USMLE STEP 3 CCS CASES

February 4, 2017 | Author: azankhan99 | Category: N/A
Share Embed Donate


Short Description

Download USMLE STEP 3 CCS CASES...

Description

USMLE WORLD CCS -20031)Location: Office Presenting complaint: A 25-year old white female presents with burning micturition. Vitals: Pulse: 80/min, B.P: 130/80 mm Hg, Temp:990F, R.R:14/min, Height: 67 inches (167.5 cm), Weight: 79 Kg (173.8 lbs). HPI: A 25-year-old personal secretary at a local office presents with the complaints of three days of burning micturition, urgency, and frequency. She had to pass urine 10 times yesterday. She also complains of suprapubic discomfort. There is no vaginal discharge, fever, hematuria, or flank pain. She has no previous history of STD or UTI. ROS is unremarkable. She has no known allergies. Medications: None. SH: She is a personal secretary at a local office. She has been married for five years and has no children. She has been smoking 10 cigarettes for the last seven years and drinks alcohol on weekends. She is sexually active in a monogamous relation with her husband. They have not been practicing contraception. Her last menstrual period was 3 weeks ago. How do you approach this case? This young female has dysuria. Her dysuria may be due to acute pyelonephritis, acute cystitis, acute pelvic inflammatory disease, acute urethritis, or acute cervicitis. You should come up with the differential diagnosis of a dysuria in a young woman. Now, perform a focused physical examination on this patient. Order physical examination: Abdominal Pelvic exam Here are the findings: Suprapubic tenderness present Normal Pelvic examination No Urethral and vaginal discharge No costovertebral angle tenderness Small discussion: This is probably a straightforward case of acute cystitis. The patient has no systemic signs of infection. So, she probably does not have acute pyelonephritis. A patient with pyelonephritis usually presents with a history of fever, chills, and flank pain. For acute uncomplicated pyelonephritis, oral ciprofloxacin for out-patients or IV ceftriaxone for hospitalized patients is appropriate therapy. The duration of treatment is usually 14 days. This patient is in a monogamous relationship with her husband and there is no history of vaginal discharge, so conditions like acute urethritis, cervicitis or acute PID are highly unlikely. A single oral dose of azithromycin or a 7-day course of doxycycline can be administered for chlamydial genital tract infections. For gonococcal genital tract infections, a single IM injection of ceftriaxone is the treatment of choice. Begin therapy for acute cystitis after confirming the diagnosis of acute cystitis by demonstrating pyuria on urine analysis. Cultures are generally not required for acute uncomplicated cystitis. Treatment with 3-day TMP-SMZ is appropriate.

The most important part of this case is whether your order pregnancy test or not. This patient is not using contraception and her LMP was 3 weeks ago. She might be a pregnant from the past 5-7 days. So, order serum pregnancy test before prescribing antibiotics. ORDERS: Urine analysis, routine Serum pregnancy test, stat RESULT: Serum pregnancy test is positive Urine appearance Turbid/ yellow pH 5.6 (normal 4.1-8.0) Specific Gravity 1.016 (normal 1.003-1.030) Bilirubin Negative Ketones Negative Glucose Negative Blood Negative Leukocyte Esterase Positive Nitrite Positive Protein Negative Microscopic Analysis: Epithelial cells Bacteria 20-30/hpf RBC/WBC Casts WBC 30-40/hpf RBC 3/hpf Crystals Mucus Review order: Oral Amoxicillin continuous (for 7 days) Followup for prenatal assessment Prenatal vitamin Consider counseling about the following: Smoking cessation Limit alcohol Regular exercise Use of seat belt Medication compliance Patient education Primary Diagnosis: Uncomplicated acute cystitis and Pregnancy Treatment of uncomplicated cystitis: Normal healthy women a 3 day TMP-SMZ Diabetic women, symptoms for >7 days, recurrent UTI, >65 yrs age group a 7 day TMP-SMZ Pregnancy a 7-day amoxicillin; if the patient is allergic to penicillin – 7-day nitrofurantoin

2) Location: office

Presenting complaint: A 75-year-old white male presents with forgetfulness. Vitals: Pulse:75/min, B.P:110/75, Temp:98.6 F, R.R:16/min, Height:72 inches (180 cm), Weight:65kg (143 lbs). HPI: A 75-year-old white male is brought to the outpatient clinic by his son with the complaint of forgetfulness for the last two years. He reports that his forgetfulness was mild initially but it has gradually worsened and now he cannot continue his routine activities of daily life. He has also developed paranoid features and accuses his son of mixing poison in his food. He eats and sleeps well, does not take any recreational drugs, smoke or drink alcohol. He has been sexually inactive since the death of his wife 15 years ago. There is no history of CAD or stroke. An older sister has a history of dementia. He has no known allergies. He takes docusate for constipation. FH: Father died of MI at 68 and mother died of breast cancer at 55. His rest of the ROS are unremarkable. How to approach this case? This patient has presented with progressive memory loss, which is most likely due to dementia. Complete physical examination to detect some occult/atypical medical illness should be performed. Neurological examination is of special consideration, which may help us detect focal neurologic deficits due to stroke; rigidity or tremors due to Parkinson’s disease. Patient with Alzheimer’s does not present with motor deficits. Perform the physical examination: General HEENT/Neck Neuropsychiatric examination Chest/lung examination Heart/CVS examination Abdominal examination Rectal examination Genital examination Lymph node examination Extremities Skin Results of the physical examination: GeneralThe patient is alert but appears poorly groomed. HEENT Thyroid gland is normal, no other abnormality found. Abdominal examinationWNL Rectal examinationNormal sphincter tone and prostate; brown colored stools with no evidence of occult blood; no palpable masses. Chest/lungsWNL CVSWNL Lymph node examination No lymphadenopathy Neuropsychiatric examinationOn Mini-mental state examination he can’t spell ‘world’ backwards, calculate, copy designs, recall objects or follow 3-stage commands. Discussion: The major dementia syndromes include Alzheimer’s disease, Parkinson’s and Lewy body dementia, vascular dementia, frontal lobe dementia, and reversible dementia. DSM-IV criteria for the diagnosis of Alzheimer’s disease: Gradual impairment of cognitive function resulting in social or occupational

dysfunction; Impaired recent memory with one or more of the following: impaired executive function, impaired visual processing, impairment of skilled motor activities; Absence of other psychiatric, neurologic or systemic diseases; Occurrence of deficits not exclusively in the setting of delirium Vascular dementia is suggested by the presence of abrupt onset of symptoms with stepwise deterioration, focal neurologic findings on examination, and presence of infarcts on CT scan. If dementia is due to Parkinson’s disease, typical features like rigidity, tremor and bradykinesia will be evident. The recurrent graphic visual hallucinations and delusions are the most characteristic feature of associated dementia. In frontal lobe dementia there is impairment of executive function, behavior is disinhibited, and cognitive function is normal or minimally abnormal. These patients don’t have insight of their problem. Many causes of dementia are reversible and they include the followings: medication induced; metabolic disorders like vitamin B12 deficiency, thyroid problems, hyponatremia, hypercalcemia; alcohol related; hepatic, and renal dysfunction; normal pressure hydrocephalus, and CNS disorders like tumors and hematomas. The American Academy of Neurology recommends routine non contrast CT/MRI of the head, vitamin B12 level, and TSH level in all patients with dementia. There are no clear evidence to support or refute ordering "routine" laboratory studies such as a CBC, BMP. and LFTs. Screening for neurosyphilis is done only when there is high index of suspicion. Test for HIV should be considered in a high-risk patient, but it is not a routine part of investigations. Thus, we will order the following tests Order routine: CBC with differential, routine BMP (Na, K, Cl, Co2, BUN, Cr, Blood glucose, Ca), routine LFTs Serum TSH Serum B12 Serum folic acid Non-contrast CT scan of head Result of Labs: CBC WBC counts of 12,000/micro-L with 85% neutrophils BUN 11 mg/dL Serum creatinine 0.7 mg/dL Blood glucose 110mg/dL Serum calcium 9.5 mg/dL Serum TSH Normal Serum electrolytes Na is 140 meq/L, K is 4.0 meq/L, Cl is 100 meq/L. LFTs WNL CT scan of head Moderate to severe cortical atrophy So based on the history, examination and lab results, the most likely diagnosis in this patient is Alzheimer’s disease. Order review: Oral Donepezil, continuous (As cholinesterase inhibitors can improve cognitive

function in patients with AD) Oral olanzapine, continuous for treatment of delusions in AD, atypical antipsychotics are preferred. The older low potency typical neuroleptics like chlorpromazine are highly sedating, and have anticholinergic side effects (May worsen memory). High potency neuroleptics like haloperidol are associated with high incidence of extra pyramidal side effects. Counsel patient: Advance directives Alzheimer’s support group Patient education Family counseling about safety plan regarding cooking, driving, and falls. Supportive care SSRI (Fluoxetine) antidepressants if depression is present Buspirone if patient has anxiety Temazepam is the drug of choice if the patient has sleep problems. Do not use short acting sedatives like triazolam as they exacerbate mental confusion. Primary Diagnosis: Alzheimer’s disease

3)LocationLocation: Emergency room Vitals: Pulse: 80/min; B.P: 145/90 mm Hg; Temp: 98.8 F; R.R: 16/min; Height: 72 inches (180 cm); Weight: 72 Kg (158.4 lbs) . CC: Severe chest pain HPI: A 60-year old white male comes to E.R with a two- hour history of severe central chest pain that began while relaxing on the couch at home. The patient denies any exertional activity prior to the onset of symptoms. The pain is constant, 9/10 in severity, crushing in quality, and radiates to the left side of the jaw and left shoulder. There is associated nausea without vomiting. Over the past two months he has experienced several episodes of exertional chest pain while at work. The pain is usually relieved with rest. He did not seek any medical attention thinking that the pain was work related muscle spasms. Medical problems include hypertension for which he has been taking hydrochlorothiazide the past 10 years. He has no known allergies. FH: His father died of MI when he was 55. Mother is 85 yrs old and healthy. SH: He has been married for 34 years and has two sons. He is not sexually active. He has a 30-pack per year smoking history. He drinks moderate amounts of alcohol on weekends, but denies the use of recreational drugs. He is a truck driver. ROS: Denies headaches, vision changes, tinnitus, or vertigo. Denies muscle tenderness, joint pain, stiffness, or weakness. Rest of ROS is unremarkable. How to approach this case? This patient has come to the ED with chest pain of recent onset which has many causes and some of them may cause sudden death. Therefore, all such patients should be transported to ED immediately. Oxygen, IV access, cardiac monitoring, and EKG need to be done as soon as possible. Aspirin is given if MI is likely. Therefore, we should order the following: Order: Continuous supplemental oxygen Oral Aspirin Sublingual nitroglycerin 0.4 mg every 5 minutes x 3 as needed for chest pain

Continuous pulse-oximetry Intravenous access Continuous cardiac monitoring Continuous BP monitoring EKG, 12 Lead, stat The history and physical examination complemented by selected tests such as chest X-ray, EKG, cardiac enzymes allow the physician to accurately diagnose most causes of chest pain, especially CAD. Therefore, we will also do the following Physical Exam: General appearance HEENT/Neck Heart examination Lung examination Abdomen Rectal exam (As this patient may require Heparin for CAD) Musculoskeletal (for possible DVT) Order: Chest X-ray, PA, portable, stat Cardiac enzymes, CK-MB and troponin-T, stat and every 8 hours x 2, Results: Chest/lungsThe chest wall is normal. The diaphragm and chest move equally and symmetrically with respiration. No abnormality is detected on percussion and auscultation. CVSNormal S1 and S2. No murmurs, rubs, gallop, or extra sounds. Pulses are normal. There is no jugular venous distension. Blood pressure is equal in both arms. EKGNormal sinus rhythm with 3 mm ST depression and T wave inversion in lead II, III and AVF. Pulse oximetryShows O2 saturation of 96% on 2-lit nasal cannula & 92% on room air. Cardiac enzymes and CXRPending Cardiac monitorNo change of vitals from the time of admission. Heme occultNegative Patient is still complaining of pain. His history, CAD risk factors such as smoking, HTN, family history, and the EKG findings of T wave inversion suggest the diagnosis of either unstable angina or non-Q wave infarction. In cases of unstable angina, troponins or CK-MB are not elevated but they are elevated in cases of non-Q wave infarcts. However, even in cases of non-Q wave infarcts, troponins levels may not be detectable at initial presentation. We will start heparin and anti-ischemic therapy in this patient. Order review: Shift to ICU PTT/PT stat IV heparin, continuous with every 6 hours PTT or Enoxaparin Q12 hours without frequent PTT monitoring IV nitroglycerin, continuous (blood pressure should be monitored as hypotension may develop) IV Metoprolol 5mg x 3 (5 minutes apart) Bed rest, complete

NPO, as this patient may require emergency catheterization Input and output charts CBC with differential, stat and daily to monitor heparin-induced thrombocytopenia Basic metabolic panel (BMP), stat and daily Results: Cardiac enzymes are within normal limits. Discussion: The guidelines for the management of USA/NSTEMI are: Bed rest with continuous ECG monitoring in patients with ongoing rest pain. NTG, sublingual, followed by intravenous administration, for the immediate relief of ischemia. Aspirin should be given as early as possible. Clopidogrel is used in patients who are unable to take ASA because of allergic reactions or major gastrointestinal intolerance. Pulse oximetry and/or ABG Supplemental oxygen for patients with cyanosis or respiratory distress IV Morphine when the chest pain is not immediately relieved with NTG or when acute pulmonary congestion and/or severe agitation is present. IV beta-blocker followed by a oral dose provided there are no contraindications. The goal of the treatment is to bring the heart rate down to 60-70/min. If there are any contraindications for beta blockers and the patient is having continuous or frequently recurring, a nondihydropyridine calcium antagonist (e.g., verapamil or diltiazem) can be used as initial therapy in the absence of severe LV dysfunction or other contraindications. Routine use of ACEI to all patients with USA/NSTEMI is a class II recommendation. However, an ACEI is used when hypertension persists despite treatment with NTG and a beta-blocker, in patients with LV systolic dysfunction and in diabetic patients. Anticoagulation with LMWH or intravenous unfractionated heparin should be added to antiplatelet therapy with ASA and/or clopidogrel. Enoxaparin is the best studied of all. Heparin should be given for at least 2 days. A platelet GP IIb/IIIa antagonist (Tirofiban or eptifibatide) should be administered, in addition to ASA and heparin, to patients in whom catheterization and PCI are planned. Early invasive therapy is indicated for high-risk patients with UA. Patients with refractory ischemia, recurrent symptoms, ST segment depression, and hemodynamic instability are at high risks. These patients should be referred for angiography and revascularization. In the absence of these findings, either an early conservative or an early invasive strategy in hospitalized patients without contraindications for revascularization. Role of statin therapy is conflicting. However, in the acute setting the mechanism of benefit from statin therapy probably involves anti-inflammatory effects rather than the lipid lowering. The other added benefit is, studies have shown that the long term compliance is better if the statins are started before the discharge. Thrombolytic therapy is not indicated in the treatment of USA/NSTEMI and should not be used. Order review: Shift to ward and continue the above treatment. Repeat 12 lead EKG Order fasting lipid panel

LFTs (for baseline before you start statins) Trans thoracic echocardiography Cardiac catheterization Obtain TSH if the patient has abnormal lipids especially elevated triglycerides. Order review: Aspirin, continuous Sublingual nitroglycerin, continuous as needed Atenolol, oral, continuous Parvastatin, continuous Patient education Cessation of cigarette smoking Limit alcohol Exercise program Medication compliance Relaxation techniques Low sodium diet Follow up visit at two to six weeks Diagnosis: Unstable angina

4)LocationLocation: Office Presenting complaint: A 24-year-old married female presents with nausea and vomiting Vitals: Pulse:82/min, Temp:98.6 F,R.R:15/min, B.P:130/70 mm Hg,Height:162cm, Weight:62 kg (136.4lbs) HPI: A 24-year-old Asian female presents with complaints of nausea and vomiting for the last several days. She feels more nauseated in the morning and also complains of breast pain. Her last menstrual period was 7 weeks ago and before that her menstrual periods have always been regular with a 28-29 day cycle. She was married 8 months ago, is sexually active with her husband, and has never been pregnant. The patient denies abdominal pain, fever or vaginal discharge. She has been a one pack per day smoker since her teenage years. She is not on any medications, does not drink or use recreational drugs. The patient migrated to the United States 5 years ago and does not recall her vaccination history. There is no history of sexually transmitted disease, but she has never been tested for STDs. Recently, she has been experiencing some constipation, other wise her bowel and bladder functions are regular. She is doing well at her office where she works as a secretary and has no emotional stresses. ROS are unremarkable. Hospitalization/ProceduresNever Other Medical ProblemsNone AllergiesNone Current MedicationsNone VaccinationsSee HPI Family HistoryFather is healthy at 55; mother is healthy at 45. Maternal grandmother died of breast cancer at 60. She has one older sister who is healthy.

Social historySee HPI Recreational historyAttending social events and watching movies How to approach this case? This young sexually active female has presented with nausea, vomiting and amenorrhea, which are most likely due to pregnancy. Therefore do complete physical examination, which should include abdominal, breast and genital examination to look for signs of pregnancy and order a pregnancy test. Here are the results: Breast examination: Mild breast tenderness bilaterally. Abdominal examination: Normal Pelvic examination: Bluish discoloration of vulva and vagina present; no vaginal discharge; no vaginal or cervical lesions, uterus is globular and soft; no adnexal masses or adnexal tenderness noted. Rest of the examination is within normal limits. Labs: Urinary beta-HCG, routine The "gold standard" for diagnosis of pregnancy is the detection of the beta subunit of human chorionic gonadotropin (hCG) by immunologic techniques in blood or urine. When performed in a clinical laboratory, the sensitivity and specificity for both blood and urine pregnancy tests are between 97 and 100 percent. Results: Urinary beta-HCG: positive Discussion: There are many signs of pregnancy, which can be grouped into presumptive, probable and positive categories. Presumptive signs are associated with skin and mucous membrane changes. The dark discoloration of vulva and vagina noted in this woman is called Chadwick’s sign. Probable signs are associated with changes in uterus. Globular shape and softer consistency of uterus is one of the probable signs. Positive signs of pregnancy are the detection of fetal heart sounds and the recognition of fetal movements. Doppler techniques enable us to detect fetal heart sounds as early as 9 weeks of gestation while the stethoscope can detect at 16 weeks. Recognition of fetal movements by an observer is possible at 20-24 weeks. Positive pregnancy testing coupled with findings on history and examination is suggestive of pregnancy. Next, confirm her pregnancy by ultrasound and start antenatal care. Ultrasound will also help us determine gestational age. A standard panel of laboratory tests should be obtained on every pregnant woman at the first prenatal visit. Additional testing of women at risk for specific conditions can augment this panel. The initial laboratory tests recommended by the American College of Obstetricians and Gynecologists are Blood type Antibody screen Rhesus type CBC with differential Basic metabolic panel Pap smear Rubella status Syphilis screen

Urinary infection screen Hepatitis B surface antigen HIV counseling and testing Chlamydia Additional laboratory tests commonly performed in at-risk individuals include: Gonorrhea Tuberculosis Red cell indices to screen for thalassemia Hemoglobin electrophoresis to detect hemoglobinopathies (e.g. sickle cell, thalassemias) Hexosaminidase A Cystic fibrosis carrier testing Serum phenylalanine level Toxoplasmosis screen Hepatitis C antibodies The first prenatal visit is a good time to discuss the patient's responsibilities and the expected course of pregnancy and delivery. Patients should be given information regarding the general plan of management for the pregnancy. Number and frequency of prenatal visits Recommendations for nutrition, weight gain, regular exercise (limited), rest, and sexual activity Routine pregnancy monitoring Listeria precautions Toxoplasmosis precautions Abstinence from alcohol, cigarettes, and illicit drugs Information on the safety of commonly used nonprescription drugs Recommendation to continue wearing seat belts during pregnancy Potential problems related to plans for travel, work outside of the home, or hobbies Childbirth classes and breastfeeding recommendations Confidentiality issues Therefore, we will do order the following in this patient Order Routine: Blood type and antibody screen Atypical antibody titer CBC with differential Basic metabolic panel Pap smear Rubella antibodies RPR Urinalysis Urine culture and sensitivity Hepatitis B surface antigen, serum Anti-HIV by ELISA, serum Transvaginal ultrasound Chlamydia, culture cervix Medications: Vitamin, prenatal, oral Iron sulfate, oral Folic acid, oral (0.4 mg)

High calorie diet Regular moderate exercise Patient education Smoking cessation Safety plan Safe sex No illegal drug use No alcohol Drive with seat belt Follow up in 4 weeks U/A and Urine culture are very important because 5-10 % of pregnant patients may have asymptomatic bacteriuria and untreated patients may develop pyelonephritis. Follow up visits: Every 4 weeks until 28 weeks Every 2 weeks between 28 to 36 weeks Every week between 36 to delivery Follow up visits: Complete physical examination Wt. Measurement Vitals especially BP Complete urinalysis Fundal measurement Fetal heart rate (110—160/min=N) measurement Glucose screening: In the United States all pregnant women will get 50 gm 1 hr glucose tolerance test between 24-28 wks gestational age. Results >135 is abnormal. If the patient has risk factors, she should be screened at 1st prenatal visit. Indications for glucose screening on 1st prenatal visit: Age> 25years Obesity Family History of DM Previous infant Wt > 4000 gm Previous still-born Previous congenitally deformed child Recurrent spontaneous abortions Advise from 2nd Trimester: Promotion of breast-feeding Childbirth classes Danger signs of pregnancy Preterm labor education Primary Diagnosis: Pregnancy 5) LocationLocation: Office C.C: A 28-year-old white male presents with bleeding per rectum. Vitals: Pulse:76/min, B.P:120/70 mm Hg, Temp:98.5 F, R.R:16/min, Height:72 inches (180 cm), Weight:72 Kg (158.4 lbs).

HPI: A 28-year-old white male presents with the complaint of having blood per rectum (BRBPR) for a week. His stools are streaked with blood. They are loose, watery, and contain mucus. He has mild colicky pain and a feeling of incomplete evacuation after defecation. He denies any history of nausea, vomiting, fever, weight loss, recent travel, or ill contacts with diarrheal illness. He has been smoking 20 cigarettes a day for the last seven years. He drinks alcohol occasionally and does not use illegal drugs. ROS are unremarkable. He has never been admitted to the hospital. He is not on any medications. He has no allergies. FH: Mother died at the age of 60 due to MI. An older brother has ulcerative colitis. Father is alive and healthy at the age of 65. SxH: Sexually active with his wife. He is a restaurant manager. Approach to the patient: Differential diagnosis of rectal bleeding include ulcerative colitis, Crohn's disease, infectious colitis, medication induced (NSAIDs, antibiotics), radiation colitis, ischemic colitis, internal hemorrhoid, anal fissure etc. The common infections include Campylobacter, Escherichia coli 0157:H7, Salmonella, Shigella etc. Consider CMV infections and Kaposi's sarcoma in an immunocompromised patient. C.difficle can sometimes present with bleeding per rectum. It should be considered in all patients who have been on antibiotics. NSAIDs can exacerbate the underlying IBD. Whenever there is visible rectal bleeding in adults an evaluation, either in an inpatient or outpatient setting is necessary depending on the degree of risk. Low risk patients (e.g. self limited rectal bleeding in an otherwise healthy young patient is most likely due to an internal hemorrhoid) can be followed as outpatient. High risk patients, such as those with acute abdomen, hemodynamic instability, or persistent bleeding need to be resuscitated and hospitalized. GI consult should be obtained as soon as possible. After assessing the stability of the patient, the next step is to determine the source of the bleeding. When the left colon is the source, the blood is usually bright red. Dark maroon bleeding or blood mixed with stool is probably from the right colon. Some times the source of the hematochezia is upper GI (in about 11% of patients) and therefore nasogastric lavage should be done in all cases to rule out an upper GI source. In this patient, who is clinically stable, there is no need for resuscitation. This patient is stable, and he is a potential candidate to have ulcerative colitis. So he needs a complete physical examination including rectal. Results of your exam: Complete physical examination is unremarkable except rectal examination, which is significant for blood stained stool. This patient is most likely suffering from ulcerative colitis based on his presenting complaints, family history and findings on examination. However, other similar conditions need to be ruled out. Discussion: Typical history coupled with characteristic findings on endoscopy establish the diagnosis of UC, which is confirmed by a biopsy. The presence of ulcerative colitis, in a first-degree relative, is an extremely important clue. Flexible sigmoidoscopy documents the extent of inflammation as well as establishes the diagnosis. Colonic biopsy is particularly helpful when the findings on sigmoidoscopy are equivocal. A barium enema can confirm the diagnosis but is usually not necessary. Colonoscopy is usually not required unless the diagnosis

is uncertain. The other problem with colonoscopy is it can cause perforation in a severely ill patient with extensive disease. Order Routine: CBC with differential, routine BMP, routine Stool examination for ova and parasites Fecal leukocytes Stool bacterial culture Liver function tests PT, PTT Flexible Sigmoidoscopy Rectal biopsy Results of labs: CBC with differential, an BMP are within normal limits. Stool for ova and parasites is negative. Stool bacterial culture shows normal flora. LFTs are within normal limits. PT, and PTT are within normal limits. Sigmoidoscopy, and rectal biopsy are consistent with the diagnosis of ulcerative colitis involving the rectum, and distal sigmoid colon. Management of ulcerative colitis: Treatment depends on the severity and extent of the disease. Topical therapy with 5-ASA compounds is the treatment of choice (not steroid enemas) for mildly active proctitis or proctosigmoiditis. They are very effective in inducing and maintaining remission. For proctitis, 5-ASA suppositories are used and enemas are recommended for proctosigmoiditis. 5-ASA enemas are significantly superior in inducing remission and have less side effects compared with steroid enemas. Although the symptomatic improvement will be seen within a few days, treatment should be continued for at least four to six weeks. Doses should be tapered off during this time as complete healing takes place. Oral therapy with sulfasalazine or with newer 5-aminosalicylates is the treatment of choice for moderately active proctosigmoiditis. Newer 5-aminosalicylates like mesalamine are more costly but have fewer side effects. Folic acid supplementation should be given to patients on sulfasalazine. Steroids are used when these 5-aminosalicylates compounds fail to induce remission. Steroids should not be used for maintenance of remission. Immunomodulator drugs like azathioprine or 6-MP are used when patient becomes steroid dependent or he is refractory to steroids. Patients with severe disease need to be hospitalized and resuscitated with IV fluids and electrolytes. They are kept NPO and given parenteral nutrition. It’s very important to monitor the patient for complications that might develop. Abdominal examination, vital signs, and plain abdominal films are used for this purpose. IV steroids are the most important treatment modality. Role of antibiotics is controversial in this patient population. However, many physicians consider giving broad spectrum antibiotics if the patient has fever, leukocytosis, or any indication of sepsis. Surgery is considered for refractory cases. Antidiarrheal agents like loperamide may be used for symptomatic treatment of diarrhea, and anticholinergic agents for abdominal cramps. Antidiarrheal agents need to be avoided in severely ill patients. Antidepressants or anxiolytics may be required in some cases. Dietary

counseling is important in all cases. ORDER REVIEW: 5-ASA suppositories Loperamide, prn Dicyclomine, prn Diet and nutrition consult Counsel about cessation of cigarette smoking and injury prevention PRIMARY DIAGNOSIS: Ulcerative colitis, mild case involving rectum and distal sigmoid colon

6) LocationLocation: Office Presenting complaint: A 28-year-old man presents with two months of abdominal pain, and altered bowel. Vitals: Pulse:72/min, B.P:130/75 mm Hg, Temp:98.70F, R.R:17/min, Height:70 inches (175 cm), Weight:70 Kg (154 lbs) HPI: A 28-year-old white male presents with the complaints of abdominal pain and altered bowel habits for the last three months. The pain is colicky in nature. It is located in the lower abdomen. The pain does not radiate, is 5/10 in severity, worsens postprandially, and relieves with defecation. For the last three months he has been suffering with diarrhea alternating with constipation. For the last three days, he has been having diarrhea. Stools contain mucus but not blood. Symptoms occur during the day and do not interfere with his sleep or work. His weight and appetite are normal. He is not taking any prescribed or recreational drugs. He smokes 10 cigarettes per day for the last 10 years and drinks alcohol only on weekends. He is currently sexually active with his wife and always uses condoms. He has no other medical problems or known allergies. He is not on any medications. FH: No H/O IBD or colon cancer in the family. SH: Married for 5 years and has a 2 year old daughter. Works at a local university. ROS are unremarkable. How to approach this case? This patient has alternating diarrhea and constipation. This may be due to infectious causes like amebiasis or giardiasis, which are very rare. Irritable bowel syndrome is another possible cause. Bowel obstruction may present with alternating diarrhea and constipation. Colon cancer may also present in this manner, but is unlikely in a 28 year old with no family history of bowel cancer. Order: A complete physical examination, including rectal, is appropriate in this setting. Results: Completely normal physical examination Now, order the following: CBC with differential, routine Serum TSH, routine BMP, routine (diarrhea is associated with electrolyte abnormalities) FOBT Stool examination for ova and parasites Stool for white cells Stool cultures

24-hour stool collection 24-hour fecal fat estimation Results of labs: WBC count is 8,000/micro-L, hemoglobin is 15.6g/dL, and platelet count is 200,000/micro-L Serum TSH is 1 micro-U/L BMP is normal FOBT is negative Stool does not contain any ova, parasites, or white cells. Stool culture – No growth 24 hour fecal fat is WNL. Order review: Counsel the patient Reassurance Lactose free diet High fiber diet Loperamide, orally, PRN Counseling about cessation of cigarette smoking, limitation of alcohol intake, safe sex practices, and driving with seat belt Follow up visit in 2 weeks Discussion: Clinical features: Irritable bowel syndrome is a diagnosis of exclusion. Abdominal pain and altered bowel habits are the most characteristic features of this entity. Abdominal pain is crampy, located in lower abdomen, aggravated by stress, and relieved by defecation. Patients with IBS have diarrhea, constipation or diarrhea alternating with constipation. Stools contain mucus but no blood. Clinical features that suggest disease other than IBS include fever, weight loss, bloody diarrhea, large volume diarrhea, nocturnal diarrhea, nocturnal pain, anorexia and anemia. Diagnosis: When a clinical presentation is typical of IBS, only a few investigations are ordered. They include CBC and routine chemistry panel in all; thyroid hormones, and stool for ova, parasites, and leukocytes in patients with diarrhea; and flexible sigmoidoscopy in patients over 40, and in patients who have persistent diarrhea. When all the fore mentioned studies are normal, symptomatic treatment is started and patient is reevaluated after 4-6 weeks. If symptoms progress, more detailed studies are warranted. Treatment: A lactose free diet and high fiber diet are considered. A diet that produces gas is discouraged. Anticholinergic drugs for abdominal pain and loperamide for diarrhea are given on a 'as needed basis'. Prokinetic drugs may be used for constipation. Benzodiazepines are used only in cases of acute situational anxiety. Behavioral treatment is a consideration for patients who have some sort of stressor. PRIMARY DIAGNOSIS: IBS

7)LocationLocation: Emergency room Presenting complaint: A 65-year-old male presents with a chief complaint of

severe breathlessness . Vitals: Pulse: 88/min , regular, B.P: 120/70 mm Hg, Temp: 100.5 F, R.R: 25/min , Height: 72 inches (180 cm), Weight: 72 Kg (158.4 lbs). HPI: A 65-year-old white male, with a two-year history of COPD, presents to the ER with an acute onset of severe breathlessness, wheezing, and chest tightness. He also states that his cough has become more severe the past two days and the sputum production has increased in quantity and become yellowish. Other complaints include fever and malaise. He denies any chest pain and does not use supplemental oxygen at home. He continues to smoke 2-5 cigarettes/day. He does not drink alcohol or use illegal drugs. He has no allergies. His current medications are albuterol 2 puffs as needed for SOB. He has no other medical problems. Vaccinations are up to date. He was admitted once in the hospital for exacerbation of COPD. REVIEW OF SYSTEMS: Skin No complaints HEENT No vision changes, epistaxis, or sore throat Musculoskeletal Easily fatigued, chronic left knee pain Cardio respiratory Frequent productive cough, wheeze, and dyspnea Genitourinary No history of STD or UTI. Neuropsychiatric Not asked Abdominal Occasional heartburn, denies nausea, vomiting, or diarrhea How to approach this case? Based on his history, this is most likely an exacerbation of COPD. Features suggestive of COPD exacerbation include increase in the severity of dyspnea, and a change in the color and quantity of sputum. To assess the severity of exacerbation, ABGs and pulmonary function tests are performed. CXR is done to rule out disorders that mimic COPD exacerbation. ECG is done to detect RVH, arrhythmia and ischemia. CBC is performed to detect polycythemia or bleeding. Serum chemistry may point towards some metabolic cause of COPD exacerbation. Sputum gram staining, culture, and sensitivity are indicated when COPD exacerbation with a purulent sputum fails to respond to empiric antibiotic treatment. PaO2 of < 60 mm Hg and/or SaO2 48 hrs may cause cyanide toxicity, especially in patients with renal insufficiency. It is not a first line medication in pregnant women. The other good alternatives to nitroprusside are IV labetalol and hydralazine. Hydralazine is the drug of choice in pregnant patients.

IV phentolamine is the drug of choice in pheochromocytoma. Esmolol is an IV beta-blocker and is effective in acutely lowering BP when used in conjunction with a vasodilator. Myocardial ischemia is an important indication for its usage. Management of hypertension varies in certain situations: Rapid reduction for blood pressure is detrimental in patents with cerebrovascular accident. These patients can be differentiated form hypertensive emergency by the abrupt onset of focal neurological findings. Patients with acute pulmonary edema are best treated with combination of nitroprusside or nitroglycerine and loop diuretic. Drugs like hydralazine or beta-blockers or labetalol should be avoided. Patients with acute coronary syndromes are best treated with IV nitroglycerine or IV labetalol or IV nitroprusside. Aortic dissection: The primary goal is to decrease both the systemic BP and cardiac contractility. The best regimen is a combination of IV nitroprusside and an IV beta-blocker either a labetalol or metoprolol. Nitroprusside alone should not be used without a beta-blocker. Rebound hypertension secondary to abrupt withdrawal of short acting sympathetic blockers such as clonidine is best treated by re-administration of the discontinued rug and if necessary with IV phentolamine. The rare causes include 1. Pheochromocytoma 2.Cocaine intoxication 3. Interaction of MAOI and tyramine containing foods can also cause hypertensive crisis. This is best treated with IV phentolamine. 10) LocationLocation: Emergency room Presenting complaint: A 7-month-old boy presents with severe breathlessness of sudden onset Vitals: Pulse: 100/min, B.P: 80/55 mm Hg, Temp: 98.70F, R.R: 40/min, Weight: 6.8 kg (15lbs), Height: 53 cm HPI: A 7-month-old boy is brought to the ER with severe cough, stridor, and breathlessness. His 6-year-old brother went to school leaving peanuts near him. Mom found the child in respiratory distress and rushed him to the hospital. There is no family history of asthma. The infant was healthy prior to this incident. Developmental milestones are being achieved at the appropriate ages. He has no allergies. Vaccinations are up-to-date. FH: Father is 32 and healthy; mother is 28 and has DM. He has one elder brother who is healthy. ROS are unremarkable. How to approach this case: This child presents with acute dyspnea due to upper airway obstruction. Stridor is one of the important clinical signs of upper airway obstruction. There are a number of causes of upper airway obstruction in the pediatric population. Etiologies vary according to the age of the patient. Careful history, and examination as well as lateral and PA chest X-rays should be done in all such patients. First General examination HEENT/Neck Chest/lungs

CVS Results of PE General examination: The baby is crying, and in obvious respiratory distress. Chest/lungs: The child is tachypneic with nasal flaring, suprasternal, and intercostal retraction. Inspiratory stridor is noted. Air entry is reduced, and percussion note is resonant bilaterally. CVS: Normal S1 and S2. No murmurs, rubs, or gallops. Pulses are normal. No jugular venous distension. Blood pressure is equal in both arms. Review orders: Start the patient on supplemental inhaled oxygen Gain IV access, stat Pulse oxymetry, stat and continuous Cardiac monitoring, stat CXR-PA/lateral views, portable, stat X-ray neck lateral views, portable, stat CBC with differential, stat Results of Labs: CBC: normal. CXR PA and lateral views: No abnormality found. Pulse oxymetry: oxygen saturation is 91 percent on room air, and 97% on 2-lit oxygen. Cardiac monitoring: no abnormality of rate or rhythm. Discussion: This child has sudden and dramatic onset of symptoms. He had peanuts in the vicinity before he developed symptoms. Based on these findings, symptoms are most likely due to aspiration of a foreign body. The next step in this case would be bronchoscopy, which will confirm the diagnosis and aid in the removal of aspirated foreign body. Before bronchoscopy, IV steroids and IV antibiotics may be used to help reduce the chances of edema and infection. Other important causes of upper airway obstruction include croup, laryngitis, epiglottitis, retropharyngeal abscess, angioedema, peritonsillar abscess, and laryngeal papilloma. Croup is common in children aged 6 months to three years and it develops insidiously as an upper respiratory tract infection. Patients with croup have a characteristic barking cough. Laryngitis occurs in children aged greater than five, the voice is hoarse, and there is no stridor. Epiglottitis is more frequent in children aged 2-6 years. There is a short prodrome, drooling is noted, and the patient feels better when leaning forward. Patients with retropharyngeal abscess are usually younger than 6 years, and they do not have stridor. The voice is muffled and they are found to be drooling. Angioedema can occur at any age, onset is sudden, and clinical features of stridor, retractions of intercostal muscles, and facial edema are found. Peritonsillar abscess occurs in children greater than 10 years of age, onset is gradual but with sudden worsening, and there is no stridor. Laryngeal papilloma is encountered in patients of ages 3 months to 3 years, onset is chronic and voice is hoarse. The majority of foreign bodies are not visible by plain films. So, a normal radiograph can never rule out aspirated foreign body in a highly suspicious patient like this. STAT orders: IV methylprednisolone, one dose IV cefazolin, one dose

Stat consult otorhinolaryngology (ENT) subspecialty for confirmation and removal of aspirated foreign body by bronchoscopy Primary Diagnosis: Foreign body aspiration 11) LocationLocation: Office Presenting complaint: A 40-year old female patient presents with lump in her left breast. Vitals: BP:130/80 mm Hg, Pulse:86/min, Temp: 98.70F, R.R:16/min, Height:162.5cm, Weight:55 kg (12lbs). HPI: A 40-year-old white female presents with mass in the upper outer quadrant of her left breast. She first noted this mass two months ago, the mass is painless and its size does not change pre or post-menstrually. There is no nipple discharge. She does not give a history of breast lumps. There is no family history of breast cancer. She is a 10-pack year smoker and drinks alcohol socially. She is married and uses oral contraceptives. The patient denies recreational drug use. Her age at menarche was 13 and her menses have always been regular. LMP was 10 days ago. She is gravida 2, para 2, one at the age of 23 and the other at 27. Both deliveries were spontaneous vaginal. Her mother is 65 and is diabetic; her father is 70 and has angina. The rest of her ROS are unremarkable. How to approach this case: This 40-year old female has presented with a lump in her breast. Careful examination of breasts and lymph nodes especially supra clavicular and axillary should be performed when any woman presents with a breast lump. First order the physical examination: General examination HEENT/Neck Heart examination Lung examination Abdomen examination Breast examination Lymphnode examination Here are the results of examination: General, HEENT/Neck, heart, lung, and abdomen examination is WNL. Breast examination: There is a 2 cm size solid, mobile, firm, non-tender mass with distinct margins, located in the left upper and outer quadrant. There is no nipple discharge. There are no skin changes. Lymphnode examination: There is no palpable lymphadenopathy. Discussion: Based on the history and examination, this patient most likely has benign breast disease. There are multiple causes of benign breast disease. Fibroadenoma: Typically a 15-30 yr old female presents with firm, painless, mobile, (breast mouse) and well-circumscribed lumps. Fibrocystic changes: Multiple and bilateral cystic breast swellings, which are noted to be particularly painful and tender premenstrually. Papillomas: C/O bloody nipple discharge (Non-bloody nipple discharge is usually benign). Duct ectasia: Presents with fever, greenish cheesy discharge, pain, and tenderness. Mastitis: Patients complain of the sudden onset of pain, fever, chills, and

local erythema, tenderness, and induration. Breast cancer: Consider risk factors first - elderly age, family history of breast cancer, early menarche, late pregnancy, nulliparity, and late menopause. On examination you have to look for the characteristics of cancerous lesion (Single, hard, immobile lumps with irregular borders and a size of more than 2cm). Diagnosis: The best way of making the diagnosis is by using a combination of physical examination, mammography, and fine needle aspiration cytology/biopsy (triple diagnosis). Interpretation should be followed as. If all three, suggest a benign lesion - Follow the patient with 3 to 6 monthly physical exam for 1 year to make sure the mass is not enlarging. If all three, suggests malignancy - Refer to definitive therapy. If any one of the three suggests malignancy - Perform excisional biopsy. Women younger than age 35: Mammogram is not useful in this age group, as the breast tissue is very dense. However, it can be done in very high-risk patients. If you find a lump and appears to be cystic, perform FNAB/FNAC. If the aspirated fluid is non-bloody, the patient can be reassured and followed in four weeks to check for recurrence. If it recurs then the patient should be referred to surgical specialty. If the fluid is bloody, send it for cytology. If the mass is not cystic, obtain an ultrasound. If ultrasound shows a solid mass, the patient should undergo biopsy (Core biopsy or excisional biopsy). Women age 35 and older: The only difference from the above age group is all these patients should undergo bilateral mammogram along with clinical exam. Management is similar to the above group. Upto 15% of palpable breast cancers will not be visualized by mammogram. So, a negative mammogram doesn't eliminate the need for biopsy in a patient with palpable mass. We will follow triple diagnostic approach here, and will perform mammography and FNAB. Order Review: Mammography, Bilateral FNAB Ask her to come with the results Results: Mammography and FNAB are consistent with the diagnosis of fibroadenoma. In this patient, the fibroadenoma is small; therefore it does not need to be excised. The patient can be followed every three to six months for one year to assess the size of the mass. Order review: Reassure the patient Follow up visit at 3 months Screening of cervical cancer by Pap smear Counsel the patient Patient education Contraception Safe sex

Smoking cessation Limit alcohol intake Safety plan Seat belt use Diagnosis: Fibroadenoma of the left breast 12) LocationLocation: Emergency room Vital signs: BP:90/60 mmHg, HR:128/min regular, Temp:100.0° F, R.R:30/min rapid and shallow C.C: Vomitings and abdominal pain. HPI: A 20-yr-old woman presents to E.R with 5 episodes of vomiting, abdominal pain, weakness and increasing drowsiness of one-day duration. During the last 2 months she has noticed increased thirst and increased urination. The abdominal pain is diffuse, 4-5/10 in severity, constant, non-radiating and there are no aggravating or relieving factors. Vomiting is non-bloody. She has no other medical problems. She has no known drug allergies. She is not on any prescription or over the counter medications. She is not a smoker or alcoholic, and denies IV drug abuse. She has a family history positive for Type 1 Diabetes Mellitus. Her father, and paternal uncle and grandfather are all diabetics. Review of systems: She denies weight changes, fever, chills, night sweats, diarrhea, constipation, skin, hair, or nail changes, blurry vision, acute bleeding, easy bruising, indigestion, dysphagia, changes in bowel movements, bloody stools, burning on urination, recent travel, ill contacts, vaginal discharge or itch, pregnancy, heat or cold intolerance, drug or alcohol use. Last menstrual period ended four weeks ago, was normal in flow and duration. How do you approach this case? First quickly examine the patient General HEENT Neck Heart Lungs Abdomen Extremities Here are the results of the exam: General: Patient is in mild to moderate abdominal pain and appears very distressed. HEENT: Very dry mucus membranes, no JVD, EOM are intact. Rest is unremarkable. Lungs: Clear to auscultation B/L. Heart: Completely normal except tachycardia. Abdomen: Soft, non tender, normal bowel sounds and no guarding or rigidity. Extremities: No edema, calf tenderness, but week peripheral pulses. Discussion: Now, make a mental checklist of differential diagnosis, i.e. 1. Abdominal pathology like appendicitis, gastroenteritis, pancreatitis, acute intestinal obstruction etc. 2. Menstrual symptoms or pregnancy related complications

3.

DKA (Based on the family history and presenting clinical features) 4. Nonketotic Hyperosmolar state 5. Alcoholic ketoacidosis 6. Drug intoxication Order the following stat: Pulse oxy, stat and continuous Oxygen, nasal canula 2 lit, continuous NS 0.9%, bolus, stat (This patient is severely dehydrated. She is hypotensive and tachycardic. So, she needs IV fluids.) NS 0.9%, continuous, stat Finger stick glucose test, stat Urine pregnancy test, stat CBC with differential, stat BMP, stat EKG, stat Serum amylase, stat Serum lipase, stat Blood alcohol, stat Blood acetaminophen, stat Urine toxicology screen, stat Abdomen KUB, stat U/A, stat Ok here are the results: Pulse oxymetry showed 96% on room air Finger stick glucose shows 600mg/dL Urine pregnancy test is negative WBC 10,000/µL and normal differential Sodium is 129, Potassium is 5.0, Chloride is 90, Co2 is 14, calcium is 8.0, and a blood sugar of 600mg/dL EKG sinus tachycardia, nothing concerning Serum Amylase - mildly elevated Serum Lipase WNL Serum alcohol not present Serum acetaminophen - Negative Urine tox screen is – Negative for substance abuse Abdomen KUB is negative for obstruction, and no intraabdominal pathology is seen U/A showed 4+sugar, but no evidence of infection How do you approach this case? So this patient most likely has either DKA or Non-ketotic hyperglycemia. The diagnosis is based on clinical features, elevated blood sugars, and increased anion gap. To confirm the diagnosis we need to order serum ketones and serum osmolality. She has pseudohyponatremia i.e. secondary to elevated blood sugars. Treatment of hyperglycemia resolves her hyponatremia. Now order: Stat serum osmolality, stat Serum ketones, qualitative, stat Give regular insulin 15 units (bolus), stat Followed by regular insulin, IV, continuous Put the patient on cardiac monitor ABG, stat Serum Phosphate levels

Serum Mg levels Here are the results: Serum Osmolality 305 Serum Ketones - high Serum Phosphate 3.2 (WNL) ABG showed metabolic acidosis, compensated by respiratory alkalosis (pH of 7.3) Review orders: Admit the patient to the intensive care unit Nothing by mouth Bed rest Vitals as per ICU protocol Strict input and urine out chart Add potassium 20 - 30 meq to each liter of IV fluids HbA1C level Follow the patient with BMP Q 2-4 hours, then Q 8-12hours, then Q day ABG Q 2 hoursx2 After 4 hrs Stop 0.9% NS and give ½ Normal saline, IV, continuous Monitor potassium deficiency and add IV potassium chloride as needed Consider antibiotics if the precipitating cause is an infection, get a chest X-ray, sputum gram stain, and culture/sensitivity; obtain blood cultures, U/A and urine cultures. Once nausea is decreased, start oral fluids. Once the patient is stabilized transfer to ward/floor. During discharge: D/C IV insulin Start Insulin, SQ Diabetic diet (Diet, American diabetic association) Diabetic teaching Consult ophthalmology Diabetic foot care consult Home glucose monitoring, instruct patient Lipid profile Age appropriate vaccination Cessation of alcohol Smoking cessation Exercise program Seat belts Follow up appointment in 10 days Discussion: Diagnosis of DKA is based on an elevated blood glucose (usually above 250mg/dl), a low serum bicarbonate level (usually below 15 mEq/L), and elevated anion gap, and demonstrable ketonemia. Both amylase and lipase are often elevated in patients with DKA by an unknown mechanism (do not to confuse with pancreatitis). Diagnosis of Hyperosmolar hyperglycemic is based on: serum glucose levels in excess of 600 mg/dl, serum osmolality greater than 330 mOsm/kg, absent or minimal ketonemia, arterial pH above 7.3, and a serum bicarbonate above 20 mEq/L. Hyperosmolar hyperglycemic state is characterized by severe fluid and electrolyte depletion due to the osmotic diuresis produced by the extreme levels

of glucose in the serum (often >1000 mg /dL). Hydration: Patients with DKA are profoundly dehydrated and foremost in the treatment of DKA is restoration of the intravascular volume. Estimates of fluid deficits in the decompensated diabetic is 4 to 10 liters (usually 5-6 liters). Initially, one to two liters of normal saline is given as bolus, followed by 500 mL/h for the first four hours followed by 250 mL/h for the next several hours. This initial management should be guided by the patient's general condition and response, with more or less fluid as indicated. After the first 3-4 hours, as the clinical condition of the patient improves, with stable blood pressure and good urine output, fluids should be changed to 1/2 normal saline at 250-500cc an hour for 3-4 hours. Ongoing reassessment is critical. Insulin: The standard insulin dose is an initial bolus of 10 to 15 units of regular insulin followed by a continuous infusion at a rate of 8 to 15 units per hour. When the glucose levels begin to approach 250 mg/dl, insulin infusions are continued, but the fluid composition is changed to include 5-10% dextrose in water to avoid hypoglycemia. Potassium: Potassium: Regardless of the serum potassium level at the initiation of therapy, during treatment of DKA there is usually a rapid decline in the potassium concentration in the patient with normal kidney function. Potassium replacement is indicated in all patients with the following features: K of 20:1 A short explanation to understand the causes: Prerenal azotemia can lead to ATN, which is the most common cause of the acute renal failure. Two basic mechanisms: a) Decreased effective intravascular volume (includes third space loss) b) Decreased cardiac output. Renal causes have specific etiologies ranging from infectious to toxin mediated and can be treated with specific etiology directed therapy. Post renal or obstructive must be promptly treated to avoid any damage to the kidney. Two most common are BPH and nephrolithiasis. Here the case scenario is of a pre renal failure most probably due to dehydration resulting from diarrhea. Order routine: Discontinue lisinopril Discontinue ibuprofen Continue rest of his current medications Transfer to the floor/ward Vitals q2h 24hr urine protein Diabetic and renal diet with 100% hand assistance (Only if the patient is awake) Complete bed rest until his mental status returns to baseline Heparin 5000 U SQ Q12 hrs to prevent DVT Renal ultrasound, routine. Stat order is placed only if you suspect an infective/renal/obstructive cause. Daily weights Strict input and output Acu checks (blood sugar), QID (4 times a day) HBA1C levels, routine Sliding scale insulin

Finally treatment: In prerenal the goal of the treatment is to increase the renal blood flow. If the patient is dehydrated continue I.V hydration. If the patient does not respond to fluids, start Lasix (furosemide) to increase the urine output. If the cause of the renal hypoperfusion is heart failure you can start dobutamine and dopamine. The use of sodium bicarbonate (type NaHCO) is rarely needed unless the patient is in severe acidosis i.e. pH of 130 with risk factors, >100 with known CAD should be treated with HMG-CoA inhibitor. ORDER REVIEW: Fasting lipid profile Glucose tolerance test Counsel the patient/Patient education Weight reduction Low fat, low caloric diet Regular exercise OCPs Pap smear Follow up in 1 week with the results Primary Diagnosis: PCOD 42) LocationLocation: Office Vitals: B.P: 140/88 mm Hg; P.R: 80/min; R.R: 17/min; Temp: 36.8C; Height: 150 cm; Weight: 80 kg. C.C: A 52-year-old woman comes to you with the complaint of sleeplessness. HPI: A 52-year-old African American woman comes to you with the complaint of sleeplessness for the past few weeks. She believes that her inability to sleep is due to episodes of excessive warmth, diaphoresis, and palpitations occurring while she’s trying to sleep. The episodes appear to be unrelated to any specific triggers, and last 3-5 minutes. These episodes occur during the daytime as well. She gets comfort in cool environment during these episodes. She had similar symptoms three months ago, which ended spontaneously. She states that she has gained weight in the last couple of months. She has been having occasional episodes of urine incontinence associated with laughing for the last couple of months. She denies diarrhea, abdominal pain, cold intolerance, and feelings of guilt. However, she is irritable about these episodes. She hasn’t had a menstrual period for the last 12 months, which was preceded by a couple of months of irregular menstrual cycles. Menarche was at the age of 13. She had two episodes of gonorrheal cervicitis, which were adequately treated. She has been sexually active, with multiple partners, throughout her life. She felt pain during her last sexual intercourse. She has been feeling vaginal dryness for the last few months. She has never been tested for HIV. Her last pap smear one year ago was within normal limits. She has hypertension for which she takes hydrochlorothiazide. She has no known allergies. Her current medications include

over the counter vitamins, and hydrochlorothiazide. There is no significant family history. She has been smoking 10 to 15 cigarettes/day for the last 16 years. She occasionally drinks alcohol on the weekends. She has not been sexually active for the last two years. She has never been married. Review of systems is unremarkable. How do you approach this case? This woman is most likely going through menopause. Her sleep difficulties are simply due to hot flashes. The diagnosis of menopause is established when amenorrhea is present for six months or more and symptoms like hot flashes, or vaginal dryness are exhibited. When there is some doubt, high FSH level confirms the diagnosis. LH levels are less helpful. This patient has hot flushes, vaginal dryness, insomnia, urine incontinence, and 12 months of amenorrhea. You don’t need to order FSH, LH in the presence of symptoms such as amenorrhea, hot flushes, and vaginal dryness. In patients who have had a hysterectomy, elevated FSH is of diagnostic value. When women develop symptoms of estrogen deficiency, estrogen replacement therapy should be started. Estrogen replacement therapy puts them at an increased risk for endometrial hyperplasia, and endometrial cancer. Therefore, endometrial thickness should be measured yearly by vaginal ultrasonography in those with endometrial thickness greater than 0.4 mm. Short-term effects of estrogen deficiency: The most frequent short-term effects of estrogen deficiency are hot flashes. Estrogen is the most effective therapy. Hot flashes may cause inability to sleep that may result in irritability, depression, and other emotional, and psychological complaints. Urinary incontinence and UTI are associated with menopause. Urinary incontinence occurs due to the atrophy of urothelium resulting from estrogen deficiency. Systemic or topical estrogen may prove effective. Estrogen deficiency results in decreased vaginal lubrication, vaginal atrophy, vaginal dryness, and dyspareunia. Estrogen is used to treat dyspareunia. Long-term effects of estrogen deficiency: Osteoporosis is a very important long-term effect of estrogen deficiency. Bone mineral density should be measured in women who are at risk for osteoporosis. Onset of dementia may be delayed by estrogen replacement therapy. All postmenopausal women should be examined annually and risk factors for heart disease, osteoporosis, and breast cancers need to be determined. This patient is stable. Order complete physical and rectal examination. Order: Complete physical examination. Results of the examination: Completely normal physical exam Discussion The patient is going through menopause. However, she needs a couple of tests to ascertain her risk for colon cancer, breast cancer, cervical cancer, osteoporosis, and heart disease. She needs a sigmoidoscopy along with the fecal occult blood test to check the risk for colon cancer, as she is over 50 years old. Order:

Pap smear Mammogram, bilateral, screening Fasting lipid Profile DEXA Scan Fecal Occult Blood Test (FOBT) Flexible Sigmoidoscopy Results: Mammogram: Normal Fecal Occult Blood Test: Negative Flexible Sigmoidoscopy: Negative Lipid Profile: Serum Cholesterol 190 mg/dl (150-240) Serum Triglycerides 98 mg /dl (35-160) VLDL 26 mg/dl (50 yr. age Presence of focal neurological findings Results: ESR is WNL CT head demonstrate no evidence of acute stroke. No midline shift. Impression: Completely normal study. Patient is still in mild pain Order: PT/INR, stat PTT, stat Results:

PT/INR and PTT are within normal limits. Order: Lumbar puncture, stat Results: Xanthochromic supernatant is noted on LP Order: Admit in ICU Continuous cardiorespiratory monitoring NPO Complete bed rest Strict ins and outs Neurochecks Q 1 hour CBC with diff, stat BMP, stat and daily EKG, 12 lead, one time (for baseline) Transcranial Doppler (For baseline) Neurosurgery, consult, stat IV fluids, NS 1000 ml bolus, followed by 50-100 cc/hr Acetaminophen with codeine po Q 4 to 6 hours prn for pain Stool softener (Docusate), twice daily Nimodipine 60 mg po QID for 21 days Omeprazole 20 mg po QD Pneumatic compression devices After you finish the orders perform complete physical examination Discussion: Based on the above findings, the most likely diagnosis in this patient is subarachnoid hemorrhage (SAH). SAH can occur even with normal CT scan. If the index of suspicion is very high, such as in patients with neck stiffness or other meningeal signs, lumbar puncture should be performed. Lysis of RBC and subsequent conversion of Hb to bilirubin occurs over a period of time and turns the CSA yellow. Presence of this xanthochromic supernatant is classic for SAH. ST, T wave changes similar to cardiac ischemia may be seen on ECG. Serum electrolytes should be obtained at the time of admission and at least once daily, as they are more prone to develop hyponatremia ('cerebral slat wasting syndrome') in the first two weeks. Vasospasm remains the leading cause of morbidity and mortality in these patients. Transcranial Doppler (TCD) ultrasound assessment of the proximal middle, anterior, posterior cerebral arteries, and basilar artery flow is very helpful in predicating the vasospasm. Treatment: Patients should be placed in ICU, and kept NPO, with complete bed rest. Stool softeners and mild laxatives are useful to prevent straining. Control the headache with acetaminophen plus codeine. Stress ulcer prophylaxis should be given with either H2 blockers or PPIs. Patients should have pneumatic compression stockings applied to prevent the DVT. Management of blood pressure is important in patients with SAH. Uncontrolled hypertension causes more bleeding. On the other hand, decreased blood pressure can cause cerebral hypoperfusion resulting in infarction. The goal is to keep the systolic blood pressure 120-140 mm Hg. IV labetalol is the drug of choice if the blood pressure is high. Cerebral hypoperfusion is treated with IV NS bolus, and vasopressors. Patients with uncontrolled blood pressure should have intra arterial and central venous line. The use of prophylactic antiepileptic medications in patients with

SAH is highly controversial. Hyponatremia should not be treated with free water restriction. IV NS with salt supplements are enough to treat hyponatremia. However, sometimes 3% hypertonic saline may be needed. Because of the risk of 'central pontine myelinolysis', hyponatremia should be corrected slowly. Acute hydrocephalus is a complication of SAH. Patients should be frequently examined, and if necessary ventriculostomy should be performed. Once the medical management has been established and the patient is stable, a standard 4-vessel angiogram should be performed. Further management is beyond the level of this exam. 52) Location: Office Vital signs: B.P: 148/90 mm Hg; P.R: 110/min, regular; R.R: 24/min; Temp: 37.0 C. C.C: Rapid heart beat and palpitations. HPI: A 48-year old white female presents to your office for evaluation of recent onset of rapid heartbeat and palpitations. They occur without warning, are regular in rhythm, and resolve spontaneously. She also reports that she has lost 12 pounds, during the last two months, despite a good appetite. She says she’s having trouble getting to sleep. She denies any constipation, diarrhea, blood in the stools, or melena. She denies any heat or cold intolerance. She has noticed decreased duration of her menstrual cycles recently, and thinks she’s reaching menopause. She denies any chest pain, dizziness, syncope, leg swelling, shortness-of-breath, orthopnea, or PND. PMH: No H/O heart disease, HTN, or DM. Nothing significant except an anxiety disorder. She has no known allergies. FH: There is no family H/O sudden death. SxH: The patient is sexually active with husband. They use condoms as contraception. SH: She denies smoking, alcohol, or IV drug abuse. Medications: None. ROS: Unremarkable. How would you evaluate this patient? Complete physical examination Results: On exam she has rapid speech. Her hands are warm and moist. Mild diffuse nontender enlargement of the thyroid is noted. Rest of her exam is completely normal. Orders: CBC with diff, stat BMP, stat ECG, 12 lead, stat Serum TSH Serum Free T3, and T4 Results: CBC with diff is within normal limits BMP is within normal limits EKG showed sinus tachycardia TSH is low - 0.08 µU/mL T3 and T4 are elevated Review order: 24-hour radioiodine uptake Follow-up with the results

Results: Radioiodine uptake is increased Review order: Propranolol, oral, continuous Methimazole, oral, continuous Follow up in 4 weeks. Advise to stop methimazole 4 days prior to follow up. Review order: CBC with diff Stop Methimazole Radioiodine, one time Follow up in one month Discussion: A 48-year old white female presented with palpitations, weight loss (despite having a good appetite), difficulty sleeping, and menstrual problems. This is one of the classic presentations of hyperthyroidism. The combination of weight loss and good appetite is classic for hyperthyroidism. Approach of a patient with palpations: A complete history and physical examination should be performed in all hemodynamically stable patients. There are no evidence-based guidelines to direct the laboratory workup on patients with palpitations. We routinely obtain 12 lead ECG, CBC with diff, BMP, and TSH levels. Identify common causes like anemia, electrolyte imbalance, and thyroid abnormalities. If the initial approach does not establish the definitive diagnosis, further evaluation is indicated. Patients are classified as either low risk or high risk depending on the risk factors. High-risk patients: Patients with H/O syncope or dizziness. Patients with a family history of sudden death, arrhythmias, or long QT syndrome. Any patient with underlying organic heart disease, which include scarring from prior MI, cadiomyopathy, significant valvular disease, and HOCM. These patients are at high risk of developing ventricular tachycardia so should be evaluated with either ambulatory monitoring or an inpatient electrophysiological study. Low-risk patients: Patients with no family or personal history of arrhythmias. No H/O dizziness or syncope. No underlying organic heart disease. These patients should be reassured. However, ambulatory monitoring (24-hour Holter monitor) is indicated if the ECG shows sustained arrhythmia. Hyperthyroidism: The most cost affective approach to a patient with suspected hyperthyroidism is to obtain serum TSH levels. A patient with normal TSH is very unlikely to have hyperthyroidism. However, it is reasonable to obtain a simultaneous free T3 and T4 if the index of suspicion is very high. If the TSH is less than 0.1 ml and free T4 is elevated, hyperthyroidism is confirmed. If the TSH, free T4, and T3 are elevated, a TSH producing pituitary tumor should be suspected and MRI of the brain should be ordered. Once the diagnosis of hyperthyroidism has been established, the underlying cause should be determined. Basically, hyperthyroidism is categorized into two broad classifications: Graves' disease and toxic multinodular goiter.

Postpartum thyroiditis, iodine induced, or factitious hyperthyroidism. Often, Graves disease is diagnosed on clinical grounds (diffuse goiter, ophthalmopathy) alone. Measurement of TSH receptor–stimulating autoantibodies (TSI), is not routinely recommended for the diagnosis of Graves disease. However, a 24-hour radioiodine uptake is necessary to confirm Graves disease and to exclude other possibilities (it will be elevated in the first group and decreased in the second group). Once the diagnosis is made, treatment should be started. Sub acute and postpartum thyroiditis are usually transient and require only symptomatic treatment. Radioiodine is the treatment of choice for people over 20 years old. Obtain a pregnancy test before you take a radioiodine uptake because it is contraindicated in pregnancy. Radioiodine causes destruction of the thyroid and often associated with a small risk of thyrotoxicosis. So, all elderly and patients with cardiac problems should be pretreated with antithyroid drugs for at least one month. Antithyroid drugs must be stopped 3 to 5 days prior to the radioiodine treatment to achieve optimum iodine uptake. Ophthalmopathy may be aggravated by radioiodine treatment. So, physicians often start tapering course of steroids at the time of radioiodine treatment if the patient has evidence of ophthalmopathy. Symptoms of hyperthyroidism can be seen upto 2 to 3 months after the treatment of radioiodine. Either antithyroid drugs or beta blockers can be used to control the symptoms during this period. Patients should be followed at 4-6 week intervals with clinical examinations, and the free T4 level (not TSH). If symptomatic hyperthyroidism persists after six months, radioiodine uptake should be repeated. Antithyroid drugs are primarily used in pregnancy, when radioactive iodine is contraindicated or in a patient of less than 20 years old. Propylthiouracil is the drug of choice in pregnancy. These are associated with granular cytopenia, so always obtain a baseline CBC with differential before starting the treatment. PTU is also associated with hepatotoxicity so, baseline LFTs are important. Finally, surgery is indicated in pregnant patients who do not respond to propylthiouracil therapy. Euthyroid state is usually achieved in two to four months. Follow the patient in six weeks with free T4 levels. Beta-blockers (eg, propranolol or atenolol) can be used if the patient is tachycardic, anxious, sweating, or having tremors. If beta-blockers are contraindicated, such as in symptomatic bronchial asthma patients, verapamil can be given.

53) Location: Outpatient Clinic, Pediatrics. Vital signs: Normal. C.C: 2-1/2-year-old girl with abdominal pain and constipation. HPI: The patient is a 2-1/2-year-old Hispanic female who presents today for evaluation of abdominal pain, constipation and anorexia that had been present for approximately eight weeks. She arrives with her parents who report that her symptoms started slowly at first with some anorexia and then slowly they began to notice that she was constipated more frequently and had complaints of abdominal pain. The parents note that the patient’s stools are normal in

appearance, other than being somewhat hard and round. She has had no episodes of rectal bleeding or melena. She had no complaints of nausea and has not been vomiting. The patient has not had regular medical care but has received immunizations at a local health department. The family lives in a house built in the 1930s. They have been remodeling the home over the past year and a half including tearing down walls and refinishing the flooring. The house still has its original windows. The father works in the construction business and mainly does the initial demolition prior to putting up a new structure. Mother stays home and watches the patient. They have a school age child who is six years of age. There are no pets in the house. They have city water. The patient herself has never had trouble with constipation or anorexia prior to this time. The patient’s gross motor development has been normal up until this point and she has been meeting her developmental milestones. The patient spends all of her time at home. She does not go outside of the home for day care. How to approach this case: This is a 2-1/2-year-old child presenting with anorexia, abdominal pain, and constipation. The differential diagnosis is rather broad and constipation is an exceedingly common problem in the pediatric population. However, this child has some red flags that might make one want to pursue a slightly different course in the workup of her constipation symptoms. First, she has not had routine health check-ups so she has not had a screening hemoglobin level to identify iron deficiency anemia nor has she presumably had a screening lead level to identify children who are high risk of lead poisoning. Both of these studies should be obtained on this patient as part of her constipation workup. Orders: Complete physical exam Results: General: The patient is well developed and well nourished, in no acute distress. HEENT: Slightly pale conjunctivae, otherwise normal. Cardiovascular, and Lungs: Normal. Abdomen: Bowel sounds are present but slightly diminished. Belly is soft, nontender, and slightly distended with stools felt in the left lower quadrant. Extremities are normal. Neurologic: Nonfocal and appropriate for age. The patient is a 2-1/2-year-old with constipation so one would want to at least treat that problem. Orders should include a bowel regimen to improve the constipation symptoms. Hypercalcemia and occult urinary tract infection may present with constipation. Orders: Fingerstick lead level (Unfortunately this is not available in CCS software; So you can directly obtain 'blood lead, quantitative'), CBC with differential, routine Basic metabolic panel, routine Calcium level, routine Milk of magnesia 10 cc QD, Docusate 20 mg QD, Urinalysis, Follow up appointment in three to five days to review laboratory studies and see if improved. Results:

Fingerstick lead of 70 mcg/dl, Hemoglobin is 10.7, hematocrit is 33, MCV is slightly low at 76 cl/cell. U/A - WNL Discussion: This patient has an elevated fingerstick lead. The current guidelines indicate that any level over 9 is considered elevated. The patient also has a mild decrease in her MCV and her hemoglobin and hematocrit are slightly decreased suggesting microcytic anemia. Both iron deficiency and lead poisoning can induce a microcytic anemia. As the first test to follow up with the patient’s elevated fingerstick lead level, one should obtain a venous blood lead level. Order: Venous blood lead level. Results: 54 mcg/dl. Discussion: This patient has an elevated blood lead level at a level sufficiently high to warrant treatment. Treatment is aimed at assessing the patient’s environment to see what sources of lead may be contaminating the patient’s environment; changing the child’s behaviors, particularly hand-mouth behavior which can contribute to the ingestion of lead dust; ensuring that the child has adequate nutrition, particularly calcium and iron to decrease lead absorption; and lowering the patient’s whole body lead level through chelation therapy. In general, chelation therapy is warranted when the blood lead level is greater than 45 mcg/dl. Monotherapy is indicated up to 69 mcg/dl; greater than 69 mcg/dl warrants two-drug chelation therapy. Orders: Home inspection for sources of lead - Type 'Lead paint assay at home' Dietary recommendations to increase calcium in the diet to approximately 1 gram of calcium per day. This may be obtained either through milk and other dairy products or calcium-fortified orange juice as well as recommendations for iron therapy since the patient has iron deficiency or simply add an iron-containing multivitamin if the patient does not have iron deficiency. Serum iron, ferritin, and TIBC Succimer (DMSA) chelation therapy, oral continuous Liver function tests, erythrocyte protoporphyrin - baseline prior to succimer therapy Follow up in one month If inspection reveals the home as the source of lead poisoning, then lead abatement (Type, 'Lead abatement agency') within the home is also a necessary part of this patient’s plan, and the patient should be removed from the home while the abatement is occurring and while the family is remodeling. Succimer, also known as DMSA, is the first drug of choice for children who have elevated lead levels in the 45-100 mcg/dl range, at a dosage of 350 mg/M2 per dose every eight hours orally for five days and every 12 hours for another 14 days. Toxicities associated with succimer include GI distress, rashes, elevated liver function tests and depressed white blood cell count. Therefore when ordering succimer further orders include CBC and hepatic panel. One should obtain these at baseline as well. Results: Patient has completed the chelation therapy without any side effects.

Iron studies were normal. Baseline liver functions were normal. Erythrocyte protoporphyrin level was elevated. Her constipation has improved and she no longer needs the milk of magnesium or docusate. Orders: Repeat lead level, (Type blood lead, quantitative) CBC, Erythrocyte protoporphyrin today. Discussion: Exposure to lead can cause subtle cognitive defects in children. Currently the accepted level for the threshold of concern is a blood lead level greater than 9 mcg/dl. Because of increased public awareness of the toxicities associated with lead, screening programs that routinely screen for lead as well as anemia have been able successfully to identify children who have increased exposure to lead and possible toxicity from it. Patient populations who are at increased risk of high lead levels include immigrant families, particularly Hispanic ones, who may use ceramic ware glazed with lead paint; children who live in poverty; who are younger than six years of age; who are African-American; or who dwell within a city. Children can have increased exposure if they live in a house or spend considerable time in a structure built before 1950 when use of lead-based paint was prevalent. Children whose family members work in areas that may have elevated lead levels (including metal refineries, battery recycling plants, maintenance workers on bridges and boats, and demolition workers) may receive “second hand” dust exposure from contaminated clothing. Other sources include window blinds, zippers, painted furniture and mineral supplements, particularly ones that have been brought to this country from a country in which lead levels are not vigorously monitored. Lead dust is a particular problem for children as it usually accumulates in places such as windowsills where paint along the window is frequently rubbed with the release of dust particles; toddlers who, because of their developmental stage, tend to mouth objects including windowsills, toys, and their hands become exposed to this dust. Because of monitoring, most children present with asymptomatic lead poisoning that is noted on screening laboratory tests. The fingerstick lead level is the initial screening test; however, because of contamination problems, this level is frequently higher than a venous blood level and therefore any fingerstick screen that is elevated should be followed up in 48 hours with a venous blood lead level. Erythrocyte protoporphyrin levels may be elevated and can be followed to see a response to chelation. It is felt that GI symptoms occur at approximately 50 mcg/dl; however, some data suggest that nearly half of children who have blood level levels in the 20-45 mcg/dl range may also have GI symptoms which may be misinterpreted. Encephalopathy secondary to lead poisoning usually does not occur unless the level is exceedingly high such as over 100 mcg/dl and this will be an indication for prompt chelation therapy. As noted above, chelation therapy is recommended for levels greater than 45 mcg/dl with monotherapy being recommended for levels between 45 and 69. There are currently four different chelating agents available in the United States for lead poisoning. They are succimer, calcium edetate, BAL/dimercaprol, and D-penicillamine. Children who undergo chelation

therapy can expect to have their blood lead level rebound about four to six weeks post chelation, presumably due to the release of lead from bone stores. Those children who have levels above 100 mcg are likely to have a rebound blood lead level greater than 45 mcg/dl which would warrant a second round of chelation therapy. Those children who have levels greater than 45 will generally have a rebound in their level to about two-thirds of what it had been prior to chelation therapy, and they may or may not warrant further chelation therapy. Therefore, it is important to do followup blood lead level measurements on these children. Order review: Followup appointment in one 4-6 weeks with erythrocyte protoporphyrin level and a blood lead level. Repeat chelation if still warranted. Continue multivitamin with iron. Continue calcium supplementation in the diet. Primary diagnosis: Lead poisoning. Note: You may get a case of lead poisoning with a different presentation. For example, child may present with fatigue, lethargy, not doing well in school; and on exam he has pallor. You order a CBC, which shows microcytic anemia and basophilic stippling etc.

54) Location: Outpatient Clinic. Vital signs: Blood pressure:137/79 supine, 124/68 erect; Heart rate: 85/min, regular; Respirations: 16/min; Temperature 38.8C. C.C: Cough. HPI: The patient is a 65-year-old white male with a past medical history significant for COPD with a 60-pack-year smoking history. He continues to smoke cigarettes occasionally, although he has recently cut back. He presents with a five-day history of increasing cough, increased sputum production and fever up to 38.7 for the last two days. He has dyspnea on exertion and currently has some mild dyspnea. He’s had decreased appetite, poor PO intake and a ten pound weight loss over the past two months. ROS: He denies any chills, hemoptysis, chest pain, pleuritic chest pain, abdominal symptoms/pain, diarrhea, constipation, blood per rectum, or melena. He denies any neurologic symptoms. The rest of his review of systems is negative. He had a similar illness approximately seven to eight weeks ago which was treated with cefuroxime and azithromycin, and the patient reports that after that course of treatment he got better and has been well for the past three weeks until the last five days when he had return of the cough, increased sputum production and fever. FH: Nothing significant. Medications: Takes albuterol puffs as needed. Allergies: None How to approach this case: The patient is an elderly man with a significant history of COPD now presenting with a second pneumonia in the course of about two months. He needs an evaluation right now of his O2 saturation, physical exam, and then an exploration into the etiologies behind his recurrent pneumonia. The suspicion for malignancy is very high given his 60-pack-year smoking history, the weight loss noted in the review of systems, and the recurrence of a pneumonia, particularly if the pneumonia is in the same place as the prior one.

Orders: Pulse oximetry Results: O2 saturation 90% on room air Order: Physical exam: HEENT/Neck, lungs, heart, abdomen, and extremities, Results: General: Elderly white male in no acute distress with temporal wasting. HEENT shows a clear oropharynx with upper and lower dentures. There is no neck lymphadenopathy. Temporal wasting is present. Conjunctivae are slightly pale. Cardiovascular is normal. Lungs: Decreased breath sounds throughout with rales present on the right upper lung fields posteriorly and decreased breath sounds in the right upper lung anteriorly. Increased anterior posterior distance on the chest with barrel chest habitus, and mild supraclavicular retractions. Abdomen: Slightly obese but otherwise normal. Extremities: There is bilateral tenderness of wrists, with nails more curved longitudinally and base of nail bed fluctuant in all fingers. Right index finger and middle finger show nicotine staining. Discussion: The patient has hypertrophic osteoarthropathy noticed on examination. It is characterized as chronic proliferative periostitis of long bones, clubbing of fingers and synovitis. It is more related with squamous and adenocarcinoma of the lungs. Symptoms of this condition may occur before the actual manifestation of lung carcinoma. As a No.1 killer Cancer in USA, it remains very important to know the different manifestations of lung carcinoma. This patient’s finding of hypertrophic osteoarthropathy is significant for lung carcinoma in the context of his recurrent pneumonia and dyspnea. Orders: Shift to hospital ward. Begin supplemental oxygen therapy at 2 lpm by nasal cannula (Type oxygen inhalation) IV access IV fluids at 100 cc an hour with normal saline Urine outputs, Q 4 hours Vitals: Every 4 hours Pulse oximetry every 4hours Activity: Bed rest with bathroom privileges Chest X-ray, PA and lateral, stat Blood cultures, stat Coughed sputum sample for gram stain, culture and cytology. CBC with differential, stat Basic metabolic panel, stat Begin antibiotic therapy with Levofloxacin (Levaquin) orally or IV after cultures obtained Albuterol and ipratropium nebulized treatments Q6 H and albuterol Q2H PRN for shortness of breath.

Results: Chest X ray shows an infiltrate in the right upper lobe with some elevation of the transverse/minor fissure anteriorly. There are no effusions. There is evidence of hyperinflation and chronic lung changes. Whenever Ca lung is suspected on the basis of clinical features and initial diagnostic tests, we need to perform advanced imaging procedures and other tests to establish the tissue diagnosis of lung cancer. CT scan of chest is done for mediastinal and pleural extension of the suspected lung tumor. For tissue diagnosis of the lung Ca following diagnostic modalities are available: Sputum cytology Biopsy of suspicious lymph nodes Flexible fiberoptic bronchoscopy: Biopsy specimens are taken when any endobronchial lesion is noted Pleural biopsy if pleural effusion is present Mediastinoscopy and anterior mediastinotomy when there is suspicion of mediastinum involvement by the tumor Transthoracic FNA biopsy under CT or fluoroscopic guidance when a peripheral pulmonary nodule is present Order review: Spiral CT scan of the chest Arrange for bronchoscopy Consult Pulmonary Medicine/cardiovascular surgery for bronchoscopy CBC/diff with basic metabolic panel daily Continue supplemental oxygen therapy Results: The patient undergoes bronchoscopic examination the following day. He tolerates the procedure well. Broncho Alveolar Lavage (BAL) samples are sent for cytology, gram stain, culture, AFB smear, and fungal culture. The patient continues to show slight improvement in his oxygen saturations and overall function with the levofloxacin therapy. His IV fluids can be discontinued. His supplemental oxygen can be weaned to room air. Results of the bronchoscopy showed an endobronchial lesion in the takeoff of the right superior bronchus. The area was biopsied and brushed. Cytology reveals malignant cells consistent with a bronchogenic carcinoma and cytology reveals small cell carcinoma of the lung. Order: Pulmonary Function Tests (PFT) Liver Function Tests (LFT) Serum calcium , stat CT of the abdomen and pelvis MRI brain with and without contrast Bone scan Consult oncology Consult radiation oncologist Quit tobacco use Supplement diet with high protein nutritional shakes Consider changing albuterol/ipratropium nebulizer to MDI (Metered dose Inhalers) Primary diagnosis:

Bronchogenic carcinoma presenting as obstructive pneumonia Discussion: Lung cancer incidence is about 3-5 per 1000 persons per year and the majority of patients are symptomatic at presentation. Local symptoms include cough (70%), hemoptysis (40%), dyspnea (40%), chest pain, hoarseness, superior vena cava obstruction, and wheezing. Systemic symptoms include weight loss, anorexia, weakness, and fever. Signs on exam include bone pain, hepatic dysfunction, lymphadenopathy, and neurological or cranial nerve involvement. Almost all patients diagnosed have constitutional symptoms, such as the case above. Lung cancers typically metastasize to bone, liver, lymphnodes, brain, and soft tissue. Unfortunately, screening with chest radiography and sputum cytology in patients at risk has not been found to decrease cancer mortality although it may detect disease at an earlier stage. Work up of suspected cases includes bronchoscopy for cytology and visualization, as well as High Resolution CT (HRCT) of the chest. If small cell lung cancer is found, then an MRI or CT of the brain, CT of the abdomen and pelvis, and bone scan should be performed in all patients because of the high incidence of micro/macro metastasis by the time of diagnosis. Bone marrow aspiration/biopsy is warranted in patients of SCLC (small cell carcinoma of the lungs) when there is cytopenia or increased LDH. This workup is also indicated in patients of NSCLC in whom involvement of the specific organs is suspected. PFT’s with diffusion capacity, spirometry, and oxygen saturations should be obtained early on. After staging has been completed, about 30-40% of patients will have limited stage disease and 60-70% will have extensive disease. 55) Location: Emergency room Vitals: B.P: 110/70 mmHg; P.R:100/minute; Temperature: 1020F; R.R: 15/minute. C.C: Fever and chills. HPI: A 54-year-old retired businessman is brought into the emergency room. Family members report that he has had a mild fever, chills, and body aches, for two days. However, this morning the patient exhibited a high-grade fever with chills, rigors, altered mental status, and a severe headache. He is nauseated and had non-bloody vomiting. The patient denies any neck pain, sensory changes in his extremities, weakness, seizures, or visual changes. His bowel and bladder functions are intact. PMH: Significant for hypertension the last ten years and has been taking atenolol 50 mg. once daily. SH: He denies smoking or drinking alcohol. He has no known allergies. FH: Nothing significant. ROS: No H/O head trauma. Rest is unremarkable. How would you approach this patient? This is a 54-year old male with a two to three day history of high-grade fever with chills, severe headache, vomiting, and altered mental status. The most likely diagnosis is either meningitis or encephalitis. It is difficult to differentiate encephalitis from meningitis, on clinical grounds alone. All patients should be treated as having meningitis, until proven otherwise. Order physical examination: HEENT/Neck CNS

Heart Lungs Abdomen Extremities Skin Results: On general examination the patient appears alert, awake, and oriented to time, place, and person. The patient appears mildly confused, and sleepy. He appears very ill. The only positive findings on exam are neck stiffness and Kernig sign. No focus of infection or other abnormalities are found. Fundoscopy did not reveal any papilledema. Orders: Pulse oximetry, stat and every two hours IV access IV NS, 100 cc/hr NPO except medications Hold his atenolol Complete bed rest DVT prophylaxis (Type 'Pneumatic compression stockings') Vitals Q 2 hours Urine output every two hours Head elevation Blood cultures, stat Urinalysis, stat Urine culture and sensitivity, stat CBC with diff, stat and Q day BMP, stat and Q day PT/INR, stat PTT, stat Phenergan, IV PRN for vomiting Acetaminophen, oral, PRN for headache and fever Once the blood cultures are obtained: IV ceftriaxone, continuous IV vancomycin, continuous Lumbar puncture, stat Send the CSF for cell count, protein, glucose, Gram stain, Fungal stain, culture, and sensitivity Results: Gram stain of the CSF shows Gram positive cocci CBC showed elevated white count with left shift BMP is normal PT/INR/PTT is within normal limits Review orders: Change the antibiotic according to the organism and sensitivities. Do not forget to stop the initial or unnecessary antibiotics. Order interim history and focused physical exam every four hours until you see improvement, then Q12 hours. Once the mentation is improved start clear liquids, then advance the diet. Order 'out of bed to chair'. D/C daily CBC with diff, and BMP when no longer required. Discussion: Basically, this patient is showing signs and symptoms of meningitis. He needs

to be hospitalized immediately because of his altered mental status. The patient should be kept NPO. Start IV with normal saline because the patient is on NPO and his diastolic blood pressure is 70, in spite of having a history of hypertension. Blood should be drawn immediately and sent for CBC with a differential, BMP, blood cultures, and coagulation studies (PT, INR and PTT to rule out the possibility of DIC and to obtain his baseline coagulation studies). Immediately after ordering these investigations, intravenous antibiotics should be started which are mostly empirical. The use of prior imaging studies, like a CT scan of the brain, is not necessary to proceed with a lumbar puncture. In a patient with a normal level of consciousness, without any focal neurological signs, a lumbar puncture can be safely performed even without prior imaging studies. We recommend starting intravenous antibiotics, even before obtaining a lumbar puncture. Antibiotic therapy for several hours, prior to lumbar puncture, will not significantly alter the CSF, WBC count, glucose concentration, or the results of culture. However, blood cultures should be obtained prior to starting antibiotics. CSF should be sent for gram stain, culture and sensitivity, protein, glucose, and cell count with a differential. If the patient has a history of seizures, with focal neurological signs, herpes simplex should be considered and empirical IV acyclovir should be started along with IV antibiotics. In all HIV patients, CSF should be sent for cryptococcal antigen assay to rule out cryptococcal meningitis. In acute bacterial meningitis, the CSF, WBC count will be elevated and red blood cells will be absent unless there is a traumatic tap. Glucose is usually low (less than 40 mg/dl) and the protein is elevated (more than 40 mg/dl). Gram stain is usually positive in 70-90% of untreated patients and culture is positive in around 80% of cases. The use of empirical antibiotics depends on the patient's age and risk factors. In infants of less than three months, cefoxitin plus ampicillin should be given. Cefoxitin covers most of the gram negatives and ampicillin is to cover Listeria meningitis. Dexamethasone has been indicated for H. influenza meningitis. Immunocompetent children of more than three months to adults age of less than 50 years should receive a third generation cephalosporin, preferably ceftriaxone plus vancomycin. Adults of more than 50 years of age and individuals with alcoholism or other debilitating illnesses should receive ceftriaxone plus vancomycin plus ampicillin (to cover Listeria). Meningitis, which develops after head trauma, or neurosurgical procedures, or in patients with neutropenia, should receive vancomycin plus ceftazidime. Ceftazidime covers gram-negative organisms, preferably Pseudomonas. Once the organism has been identified on gram stain, antibiotics should be directed against a specific organism. If you find gram-negative bacilli on the gram stain, ceftriaxone again is the drug of choice. If you find a Pseudomonas on the gram stain and the culture, the drug of choice is IV ceftazidime. If you find gram-positive cocci in clusters (staphylococcus), IV nafcillin is the drug of choice. First generation cephalosporins should not be used for staphylococcus infections because they do not have high permeability into the CSF. IV vancomycin is the drug of choice for penicillin allergic patients and methicillin resistant Staph aureus. If the Gram stain shows Haemophilus influenza, IV ceftriaxone is the drug of choice. If the patient is having meningococcal meningitis, the patient should be placed in respiratory isolation

and the patient can be tested for terminal component complement deficiencies (C6-C9). If you identify Listeria monocytogenes and the patient is an immunocompromised or undergoing dialysis, IV ampicillin plus IV gentamicin should be given for at least three to four weeks. Usually the period of antibiotic doses is in between 10-14 days of intravenous antibiotics. Primary diagnosis: Bacterial meningitis

56) Location: Office Vitals: B.P: 140/80 mm Hg; P.R: 88/min; R.R: 18min; Temperature: 38.1C. C.C: Pain and swelling of the right leg. HPI: A 38-yr old white female, who is otherwise healthy, presents to office with two to three day history of pain and redness of the right leg. The pain continues despite applying heat and elevating the legs. There is a mild fever today. She denies any trauma, tick or insect bites, joint pains, or prior episodes of similar problems. She has no other medical problems except menstrual abnormalities. Her gynecologist placed her on oral contraceptive pills. FH: Her mother has a H/O rheumatoid arthritis. SH: She smokes less than half a pack of cigarettes per day. Drinks alcohol on the weekends. SxH: Sexually active with her husband. ROS: unremarkable. How do you approach this case? Consider differential diagnosis: Deep vein thrombosis Cellulitis/Lymphangitis Superficial thrombophlebitis Rupture of the Bakers cyst Hematoma Order physical examination: General HEENT/Neck Lungs Heart Abdomen Extremities Skin Results: Exam is remarkable for a palpable cord, edema, warmth, and superficial venous dilation of the right lower extremity. There is no source of infection noted on careful examination of foot. Rest of the exam is unremarkable. Order: Transfer to ER Pulse oximetry, stat Compressive ultrasonogram of the right leg (Type 'Venous doppler, lower leg'), stat D-dimer, stat CBC with diff, stat Results:

Pulse oximetry shows 95% on room air USG shows deep vein thrombosis of the popliteal vein CBC shows Hb of 14, WBC of 12,000 with no bandemia, and platelet count of 220,000. Order: Rectal exam FOBT, stat PT/INR, stat PTT, stat D/C Oral contraceptive pills Results: Rectal exam is normal FOBT is negative PT is 13.6; INR is 0.9 PTT is 24 Order: LMWH (Enoxaparin), stat and Q 12 hours, sub cutaneously Acetaminophen and oxycodone for pain as needed Anticoagulation teaching (Type 'patient education') No smoking Discharge to home with follow up in office next day Review: Brief physical exam and interim history Coumadin/warfarin (5 to 7.5 mg) oral, continuous PT/INR, every day until the INR is therapeutic Appointment with anticoagulation clinic to follow PT/INR (Option is not available in software, so you have to follow every day with PT/INR) Platelet count, in day 3 and 5 of heparin treatment Discussion: DVT is classified into proximal vein and calf vein thrombosis. This classification is important because proximal vein thrombosis is often associated with embolic phenomena (Board question). Even though contrast venography is the gold standard test for diagnosis of DVT, it is not recommended for screening purposes because it is invasive, associated with patient discomfort, and technically difficult. The two commonly used noninvasive tests for the diagnosis of DVT are compression ultrasonography and impedance plethysmography. Studies have shown that compression ultrasonography was superior to impedance plethysmography in guiding therapeutic strategy in patients with DVT. However, impedance plethysmography is the preferred test for the evaluation of suspected recurrent DVT because it normalizes more quickly after a previous episode than compression ultrasonography. Measurement of D-dimers is useful in excluding thromboembolic phenomena because of its high negative predictive value. However, the presence of elevated D-dimer alone cannot establish a diagnosis of DVT. D-dimers should be correlated with clinical probability, and noninvasive tests in guiding the diagnosis. Simple DVT can be managed as an outpatient. Low molecular weight heparin (LMWH) is the treatment of choice for acute DVT. Enoxaparin is the most commonly used LMWH. Warfarin can be started within 24 hours. PT/INR should be measured daily until the therapeutic range (2.0 to 3.0) is reached. Heparin is stopped in day five or six if the INR is therapeutic for at least two

consecutive days. A platelet count should be obtained on day three and five to monitor HIT (heparin induced thrombocytopenia). Heparin should be stopped if there is a >50% reduction in platelet count or a total count of less than 10 mm) hypermobile vegetations, which can potentially cause septic embolism. In patient’s with suspected infective endocarditis one should aim therapy at the most likely organism. Staph aureus is the most common organism isolated in this setting of IV drug use. The patient should be screened for other signs of endocarditis including a urinalysis which may show hematuria, chest X ray which in this case was suspicious for septic pulmonary emboli which can be seen more often in the setting of tricuspid valve endocarditis in IV drug users. Transesophageal echocardiography has become an important mode of helping to diagnose infective endocarditis and help guide management. There are a number of complications of infective endocarditis, especially with left sided disease that should be monitored for vigilantly. These are mainly embolic in nature and include CNS embolus with stroke-like syndromes or subtle neurologic defects. Emboli to the kidney may cause focal glomerulonephritis which induces hematuria or renal failure may ensure secondary to diffuse proliferative glomerulonephritis. One may see arrhythmias including various degrees of heart block and pericarditis, myocarditis or myocardial abscess. Heart failure as noted above in the indications for surgery is also a potential major complication of infective endocarditis. This patient managed to avoid most of the complications possibly because of early presentation and early treatment of his endocarditis. In patients in whom one suspicious of major complications, it could be appropriate to obtain CT scans of the head, chest, abdomen, and pelvis looking for other sites of embolic disease or infarction. One should monitor as

well renal function for evidence of kidney failure secondary to glomerulonephritis or infarction or emboli. The patient should receive four to six weeks total of antimicrobial therapy directed at the results of the blood cultures obtained. In this case with Staph aureus optimal therapy is with the penicillinase resistant penicillin, nafcillin, 2 grams IV Q4H. He also received gentamicin for three to five days initially. In patient’s intolerant to nafcillin an appropriate substitute antimicrobial therapy would be cefazolin with or without gentamicin. In patients who have allergies or who have methicillin resistant Staph aureus, vancomycin would be the agent of choice.

62) Location: Office Vitals: B.P: 130/76 mm Hg; H.R: 130/min, irregularly irregular pulse; Temp: 38.3C; R.R: 18/min. HPI: A 60 yr white female who has known H/O CAD, S/P CABG presents to your office with 2-day H/O dizziness, light-headedness, and palpitations. She describes the palpitations as irregular, and almost continuous. She denies any chest pain, angina, SOB, orthopnea, PND, or syncope. She also felt little warm since one day. She denies any cough, URI symptoms, dysuria, abdominal pain, and leg swelling. Her ROS is positive for frequency of urination. PMH: She had undergone 3 vessel CABG 3 years ago after an acute anterior wall MI. Her other medical problems include HTN, Type II DM, hypercholesterolemia, osteoarthritis, COPD, and gout. All: She has no allergies. SH: She quit smoking after her CABG. She occasionally drinks alcohol. She lives with her husband at home. FH: Father died at the age of 70 with MI. Mother died at the age of 68 from stroke. She has one brother and one sister both have HTN, and DM. Meds: She takes ASA 81mg po qd, simvastatin 20 po qhs, lisinopril 5 mg po qd, SL NTG prn, glyburide 5 mg po QD, metformin 850 mg po bid, albuterol puffs prn, and acetaminophen with codeine for osteoarthritis. How do you approach this patient? Order physical exam: General HEENT/Neck Lungs Heart Abdomen Extremities Rectal exam with FOBT Results: HEENT/Neck is WNL. There are few rales and decreased breath sounds noted at left lower base. Heart exam is WNL. Abdomen is WNL. No edema or JVD noted. Hem negative for stools. Order review: Pulse oximetry, stat IV access 12 lead EKG, stat Results: 94% on room air EKG showed atrial fibrillation with rapid ventricular response at a ventricular rate of 120-140/min. There are Q waves in anterior leads, consistent with old

MI. LVH pattern is noted. Order review: Cardizem IV, bolus CBC with diff, stat BMP, stat Chest X-ray, PA and lateral, stat CK MB, and troponin T/I , stat and Q 8hours x 2 U/A, stat LFTs, stat TSH, stat Free T4, routine PT/INR/aPTT, stat Order review: Admit to floor/ward Telemetry Vitals Q 4 hours Pulse oximetry Q4 hours Order 'old records' Diet: Consistent carbohydrate diet Activity: Bed rest with bathroom privileges Labs: HbA1C, stat Accuchecks QID(4 times a day) 2D-echo, routine Meds: Continue all home medications: ASA 81mg po qd, Simvastatin 20 po qhs, lisinopril 5 mg po qd, SL NTG prn, glyburide 5 mg po QD, metformin 850 mg po bid, albuterol prn, and acetaminophen with codeine for osteoarthritis Start Cardizem (diltiazem), IV drip, Start Heparin, IV, continuous PTT every 6 hours Daily CBC with diff Call me when lab results available Results: CBC with diff showed a WBC count of 12,000 with 3% bands. Hb is 13.5. Platelet count is 230,000. BMP showed a Na: 140, K: 4.0, CL: 102, Co2: 22, BUN: 20, Cr: 1.0. Chest X-ray showed small left pleural effusions unchanged from previous 1 yr X-ray. TSH is 1.5. Free T4 is WNL. LFTs are WNL. HbA1C is 7.2. U/A showed positive esterase, 50 WBC, and many bacteria. Urine culture is pending. PT/INR is 14.0/0.98. PTT is 30. First set of cardiac enzymes - negative. 2D -echo showed normal LV function with an EF of 50%, mildly dilated left atrium, normal valves, and mild hypokinesis of the anterior wall. Findings unchanged from previous echo. No LV/LA thrombus notified. Order review: Urine culture and sensitivity Bactrim PO QD (TMP-SMZ) Examine the patient in 2 hours After 2 hours: Interim history Monitor telemetry strip: HR is now 90-100/min; patient is still in atrial fibrillation

Repeat EKG: HR is now 90-100/min; patient is still in atrial fibrillation. Call me when needed. Examine the patient in next 6 hours Again order interim history and monitor telemetry strip Once the HR is less than 80 D/C Cardizem drip Start Cardizem PO, continuous Next day Start Coumadin po continuous Daily PT/INR Examine next day: Check CBC, PT/INR, telemetry strip Once the PT/INR is above 2.0, D/C IV heparin Discharge the patient Patient education Out patient followup in 3 days with repeat CBC, PT/INR Discussion: The principle issues in managing a patient with atrial fibrillation with rapid ventricular response include: 1. Rhythm control or rate control 2. Anticoagulation to prevent systemic embolization 3. Correcting the underlying abnormality Rhythm control: It is indicated in: 1. acute atrial fibrillation (less than 48 hours duration), 2. Hemodynamically unstable patient, 3. patients with acute coronary syndromes, 4. Patients with severe heart failure. It can be done by either DC cardioversion or pharmacologic cardioversion. DC cardioversion is particularly indicated in unstable patients. In stable patients, and patients with a reversible underlying problem can be dealt with either electrical or chemical cardioversion. The commonly used drugs for rhythm control include dofetilide, ibutilide, and to a lesser degree amiodarone. Amiodarone is particularly useful in patients with left ventricular dysfunction. Without chronic antiarrhythmic therapy, only 20-30% of patients who are successfully cardioverted remains in NSR for more than one year. The 2 commonly used medications for the maintenance therapy are amiodarone (patient with left ventricular dysfunction) or sotalol (in patients with CAD). Rate control: The 3 most commonly used AV nodal blockers for the rate control are beta-blockers, calcium channel blockers, and digoxin. Digoxin is particularly indicated in patients with heart failure or hypotension. In most other situations digoxin is less effective than a beta-blocker or calcium channel blocker. The choice between a CCB or beta-blocker depends upon physician preference, and the patient presentation. Beta-blocker is preferred in patients with H/O angina, acute MI. The use of calcium channel blockers is preferred in patients with chronic lung disease. In most situations Cardizem (diltiazem) is the preferred drug as it is easy to administer in the form of IV drip and the dose can be titrated for a goal heart rate. Patients who fail to respond with pharmacologic treatments require EP study and radiofrequency AV nodal-His bundle ablation. Choosing between Rate and rhythm control: Until recently, rhythm control has been the preferred method over rate control

for patients presenting with the first few episodes of atrial fibrillation. The thought was controlling the rhythm causes low frequency of embolic events. However, the 2 major clinical trials (AFFIRM, and RACE) have demonstrated no significant difference between the 2 groups in terms of embolic events, functional status, or quality of life. Thus, both rate and rhythm control are acceptable approaches and both require anticoagulation. There is a growing support for rate control. Anticoagulation: There are 3 situations where you should consider anticoagulation: 1. Chronic AF, 2. Recurrent AF, 3. Prior and after cardioversion. AF of more than 48 hours or unknown duration requires at least 3 to 4 weeks of warfarin prior to and after cardioversion. The target INR is 2.5(2.0-3.0). Patients with recurrent or chronic AF should be treated with long-term anticoagulation even if they are in sinus rhythm. Patients who have underlying rheumatic valvular disease, severe LV dysfunction, or recent thromboembolism should receive anticoagulation even if the duration of AF is less than 48 hours. Patients with AF of less than 48 hours duration without concurrent valvular disease, or severe LV dysfunction, or H/O thromboembolism are treated with cardioversion under IV heparin coverage but without long term coumadin. The other alternative approach for cardioversion to avoid prior prolonged anticoagulation is TEE guided cardioversion. When should I admit a patient with AF? Low risk patients (patients without valvular disease, or severe LV dysfunction) with AF of less than 48-hour duration and uncomplicated clinical status can be cardioverted and discharged from the ER. Hospitalization is necessary in high risk and hemodynamically unstable patients. Searching for the underlying cause: The common causes of new onset AF include heart failure, acute coronary syndromes, PE, HTN, hyperthyroidism, and infections. Serum TSH and free T4 should be checked in all patients even if they do not have symptoms of hyperthyroidism as there is a 3 fold increase of AF in patients with subclinical hyperthyroidism (Low TSH and normal free T4). If the AF is caused by an underlying problem, cardioversion should be postponed until the condition has been successfully treated. However, anticoagulation should be started.

63) Location: Emergency room Vital signs: B.P 80/40 mm Hg; P.R: 130/min; R.R: 30/min C.C: Chest pain from a severe motor vehicle accident (MVA). HPI: A 61 year old man involved in a motor vehicle accident (MVA), brought to the ER immediately. He complains of severe chest pain, 10/10, and non radiating. He also C/O shortness of breath. Chest wall impacted the steering wheel. No other history is available. How do you approach this patient? Order:

IV access Pulse oxy, stat Oxygen, inhalation Order general, lungs, and heart exam Results: Patient is in severe chest pain, his extremities are turning blue. Lungs are CTA B/L. There is a 15 cm JVD. S1, S2 muffled. No murmurs heard. Pulse oxy shows 88% on room air. Order review: IV NS bolus, and continue at 150 cc/hr Elevate the patient legs Continuous cardiac monitoring Pericardiocentesis, stat Results: Patient started improving His BP came upto 100/60 mm HG HR decreased to 90/min Order review: Stat EKG, 12 lead Chest -X ray, portable Transthoracic Echocardiogram (TTE), stat Pericardial fluid for cell count, stat ABG, stat CVTS (Cardiovascular thoracic surgeon) consult, stat Results: 12 lead EKG shows sinus tachycardia, low-voltage QRS complexes, and electrical alternans. CXR showed globular heart with air fluid level in pericardial cavity. TTE - Revealed fluid in the pericardial cavity. Impression: Cardiac tamponade If the CVTS doesn't want to operate, then the patient management will be as follows: Shift the patient to ICU Continue continuous cardiac monitoring Swan-Ganz catheter, stat Diet - NPO Complete bed rest Place a Foley catheter Urine output Q 2 hours Pneumatic compressions of the legs CBC with diff, stat Basic metabolic panel, stat PT/aPTT, stat Continue NS @ 150 cc/hr Gastric prophylaxis - Omeprazole (20mg) orally, once daily Type and screen for 2 units of blood Acetaminophen + codeine for pain, continuous (actually as needed in real life) Note: Transfuse blood if the Hb is less than 8 in a patient with no active bleeding, less than 10 in actively bleeding patient. Each unit of blood will increase the Hb approximately 1 gm%. Next day: D/C Foley catheter

Repeat TTE Repeat Chest -X-ray Note: If you do this much, your case will end in the exam. Further management is complicated, it is based on the patient condition, CVTS recommendations, etc. Explanation: Cardiac tamponade is a life threatening condition and should be diagnosed and treated emergently. The diagnosis of tamponade is primarily clinical. Myocardial rupture in patients with trauma usually manifests itself as cardiac tamponade. The classic description of cardiac tamponade is Beck’s triad: Hypotension (100%), distended neck veins, and muffled heart sounds. The other useful findings are tachycardia, elevated central venous pressure, pulsus paradoxus, and cyanosis of the head, neck, arms, and upper chest. Tamponade should be suspected in any patient with chest injury whose hypotension do not respond to fluids or out of proportion to the apparent blood loss. Differential diagnosis in patients with trauma should include tension pneumothorax (decreased breath sounds, deviated trachea, hyperresonance to percussion), right ventricle contusion/failure, superior vena cava obstruction, ruptured tricuspid valve, and aortic dissection. Pulmonary embolism, pericarditis, and cardiogenic shock should be considered in patients without trauma. Emergency pericardiocentesis is a potentially life-saving procedure performed in the ED. Emergent thoracotomy is indicated when the patient does not respond to pericardiocentesis and has rapidly deteriorating vital signs or cardiac arrest. After pericardiocentesis, intrapericardial catheter is left in place and attach it to a closed drainage system. Drain should be checked regularly for reaccumulation of fluid. Pericardial fluid should be sent for cell count initially and periodically (Q 24 hours) to diagnose an impending bacterial catheter infection, which could be catastrophic. If the WBC count rises significantly, the pericardial catheter must be removed immediately. A Swan-Ganz catheter is very useful to monitor the central venous pressure and it can be left in place for continuous monitoring and to assess the effect of reaccumulation of pericardial fluid. Patients should have a repeat echocardiogram and chest x-ray within 24 hours.

64)Location:ER Transient ischemic attacks (TIAs) are focal neurologic deficits caused by retinal or cerebral vascular disease. They last less than 24 hours. Crescendo TIAs are defined as two TIAs within 24 hours, three within 3 days, or four within 2 weeks. Acute TIA is defined here as a TIA that occurs within 2 weeks of presentation to a physician. Diagnostic Tests There is no routine, standard laboratory evaluation of patients with TIA because the individual medical history and specific characteristics of the TIA influence the optimal sequence and extent of diagnostic testing. 1. Duplex Ultrasound of the carotids may be helpful for those patients: whose symptoms are attributable to ischemia in a distribution of the anterior (carotid) circulation. who are candidates for endarterectomy of the carotids, where benefits of the surgery are likely to outweigh the burdens. If ordered, duplex ultrasound should be obtained within 3 days of presentation of an acute TIA (Grade C). 2. Echocardiography should be considered for those patients whose TIA is thought to be cardiogenic. This study usually does not need to be performed on an emergency basis. If a cardiac origin of an embolus is suspected, the patient can be treated with heparin anticoagulation without an echocardiogram. The diagnostic yield of echocardiography in a patient over the age of 40 without cardiac murmur or evidence from the history/physical

examination to suggest cardiac disease is so low as to make the test usually unnecessary. When cardiac disease is strongly suspected and external 2-D echocardiography is not diagnostic, additional yield may be obtained by transesophageal echocardiography. 3. CBC, Creat, ESR , glucose, PT/PTT, RPR/MAHTP, ECG - the studies could be obtained within 24 hours. 4. Imaging Studies - Brain CT scan 5. Additional Tests / In selected cases the following additional tests may be appropriate: Anticardiolipin Antibody, Antithrombin III, Protein S, FBS, Lipid profile, Telemetry/Holter Monitor Selective, Digital, or Magnetic Resonance Angiography Magnetic Resonance Image of Brain 6. Initial Medical Management of TIA 1.Blood Pressure Control. We encourage the use of antihypertensive agents that work by peripheral action, are short acting, and are unlikely to drop the blood pressure precipitouslly. 2.Anti-platelet agents. The antiplatelet agents aspirin and ticlopidine are both beneficial in the prevention of stroke following a TIA . Aspirin dose of 325 mg/day Patients who do not respond to or tolerate aspirin are candidates for ticlopidine 250 mg bid . Patients offered ticlopidine must be willing to accept the supervision associated with the use of this agent. Plavix / Clopidogrel 75 mg/day is safer than Ticlopidine. The combination of aspirin 325 mg and the antiplatelet agent dipyridamole 200 mg (Aggrenox) 7. Major Contraindication to TPA: Recent bleeding, recent trauma, recent CPR, recent operation, history of bleeding, BP more than 180/110 8. Most common cause of sudden stroke: emboli 9. Most common source of Emboli is the heart. 10. Location of stroke: ACA = legs MCA = face ICA = vision Lentico Striate Artery = Pure Motor Pons = MLF (Ipsilateral Adduct failure) = Lateral (7 TH nerve Bells Palsy). Vertebral artery = Tongue = Medial = Horner = Lateral.

65)Location: Clinic Affects 15% of people over age 65. Occurs in Down's syndrome pts at younger ages (30-40). Gradually progressive, neurofibrillary tangles. Alzheimer's Disease: senile degenerative dementia (50%-90 of dementia pts) - Loss of cortical tissue (cerebral atrophy) with increased senile plaques. Criteria for the clinical diagnosis of probable Alzheimer's disease * Dementia established by clinical examination and documented by the Mini-Mental State Examination, include: Blessed Dementia Scale, or some similar examination and confirmed by neuropsychologic tests. Deficits in two or more areas of cognition.

* Progressive worsening of memory and other cognitive functions * No disturbance of consciousness * Onset between ages 40 and 90, most often after age 65 * Absence of systemic disorders or other brain diseases that could account for the progressive deficits in memory cognition Lab. Orders: CBC, Lytes, TFTs, PRP all normal. UA toxicology -ve. CT: Evidence of cerebral atrophy with progression documented by serial observation MRI shows changes highly suggestive of Alzheimers - tangled spaghetti patches. Certain dx: not till autopsy - on PM see structural changes, abnormal proteins in brain biopsy. See shrinkage < neurons in cognitive areas of brain. Early Signs: subtle loss of memory. Person neglect, ADL. Gradual loss continues. Loss of communication skills. Later: ultimate loss of short and long term memory. Normal life span. May have good physical health. Med Intervention: No real medical therapy. Nursing support primary. Med Rx: Donepezil (Aricept) 5-10 mg tablet /day Tacrine (Cognex) Not a cure. Does not appear to stop progression as was hoped. Acts to increase amount of acetylcholine in brain to improve memory. Helps to improve in a minority of patients. Side effects: Hepatic failure, GI, abd. Pain, skin rash. Rivastigmine tartrate (Exelon) 6-12 mg cap/day Premarin for ladies Multivitamins 1 tb qd po Aspirin For vascular dementia Other supportive med Rx therapy for agitation: antidepressants, antipsychotic, sleeping aids. Nursing focus: Safety, Help maintain function as long as possible, Care for caregiver. Continuing Care: Medicare doesn't cover custodial long term. Must become impoverished to go on medicaid. Few families able to cope with entire trajectory of the illness. Nursing home care essential for some. As for Alzheimer: Remember, on the exam, when ever you counsel it takes 5 minutes for it. Social services consult - counsel, no driving - counsel, advance living will - reassure patient/family - counsel medical alert bracelet

66)Location: ER Amitriptyline: Elavil, Nortriptyline: Pamelor, Imipramine: Tofranil. The morbidity and mortality caused by tricyclic antidepressant (TCA) overdose are well recognized. Among newer antidepressants, the selective serotonin reuptake inhibitors (SSRIs) are thought to be safer in overdose. At therapeutic doses tricyclic antidepressants block the reuptake of norepinephrine, serotonin, and dopamine Their mechanisms of toxicity include: A. Neuromuscular reuptake inhibition B. Membrane depressant effects on sodium channels of the myocardium C. Alpha blockade D. Central sympathetic reflex inhibition CLINICAL SYMPTOMS: ANTICHOLINERGIC: tachycardia, mydriasis, hyperthermia, agitation, hypertension, flushed skin, decreased GI motility, urinary retention CARDIOVASCULAR:

Dysrhythmias- sinus tachycardia, AV block, Torsades, V-tach, Vfib Conduction abnormalities- QRS widening, prolonged PR interval, QT prolongation Hypertension- usually short-lived and minimal Hypotension- may be prolonged and severe CNS- agitation, disorientation, ataxia, confusion, lethargy, seizures, coma HEENT- mydriasis, blurred vision, dry mouth, nystagmus PULMONARY-pulmonary edema, ARDS, aspiration pneumonitis LEVELS/RANGE OF TOXICITY: In adults, 10-20 mg/kg is considered a moderate to serious exposure where coma and cardiovascular symptoms are expected. Approximately 35 mg/kg is thought to be a lethal dose without medical intervention. CV toxicity: Sinus tachycardia- supportive measures only Wide complex dysrhythmias Sodium bicarbonate 1-2 mEq/kg IV bolus followed by maintenance infusion to maintain pH 7.4-7.5 Hyperventilation-reserved for conditions where administration of large quantities of sodium bicarb would be contraindicated (pulmonary edema, cerebral edema, head trauma, poorly controlled CHF) Ventricular Arrhythmias Sodium bicarbonate- same dosing as above Lidocaine 1-1.5 mg/kg bolus over 1 minute then 2-4 mg/min infusion Hypotension: Fluids and Norepinephrine 8-12 ug/min (generally high dose is necessary) CNS toxicity Seizures: Diazepam: Adults: 5-10 mg over 2-3 minutes, may repeat every 10-15 minutes (maximum of 30 mg total). Peds: 0.25-0.4 mg/kg to maximum 10 mg- maximum 5mg if child less than 5 y.o. Lorazepam: Adults: 1-2mg over several minutes Peds: 0.04 mg/kg May try barbiturate if patient fails to respond to benzodiazepines Enhanced Elimination Reduce absorption: AC/C for all patients- recent studies suggest that activated charcoal is as effective as AC/lavage and AC/lavage/AC. Lavage may be indicated within 1 hour of ingestion or in patient who arrives with decreased level of consciousness and unknown time of ingestion. Increase elimination: MDAC has been shown to enhance the rate of elimination- 25 grams q 4hr following the 50g initial load (sorbitol with every 2-3 doses) Disposition Admit any patient with a QRS> 100msec. Admit any patient with a seizure. Admit any patient in need of psychiatric or medical support. Patients who have been decontaminated, who never seize or develop abnormal EKG's (other than sinus tachycardia) can be safely discharged after 6 hours of observation if otherwise stable. Patients with significant symptoms or any EKG changes, or persistent mild symptoms should be admitted and monitored until CNS returns to baseline and normal EKG for 24 hours.

67) Location:Office A young man with fatigue and jaundice. Transmission: Hepatitis A virus (HAV), the most common cause of viral hepatitis worldwide, is an RNA virus transmitted exclusively by the fecal-oral route. Its prevalence is as high as 95% in underdeveloped countries with poor sanitary conditions. The prevalence is >50% in persons older than 50 years in the United States, Canada, and Europe but falls to less than 10% in persons younger than 20 years. Therefore, many persons traveling to endemic regions do not have natural immunity and should receive passive immunization with immunoglobulin before travel. Diagnosis: The presence of active HAV infection is detected by a positive IgM anti-HAV antibody; the presence of the IgG antibody indicates previous infection and immunity. Signs and Symptoms: Fever (60%); unusual with HBV and HCV Malaise (67%), Fatigue (major complaint in HCV) Nausea (80%) ,Vomiting, Anorexia (54%) Jaundice (in adults, 62%); 66% of HCV anicteric Dark urine (84%) Abdominal pain (56%) Headache Meningismus (occasional) Management: (Supportive measures are the only treatment necessary in most cases of acute HAV infection.) Rest and balanced diet Symptomatic relief medications Family members: hepatitis evaluation & passive immunization with immunoglobulin 0.02 mL/kg IM Serial follow up of liver function tests In patients with severe cholestasis, a short course of prednisolone (30 mg/day with a taper) may reduce the severity of symptoms such as pruritus and malaise and reduce the serum bilirubin level. Immunization: Active Immunization: Hepatitis A Vaccine x 2 doses about 6-12 months apart. A single dose of a highly purified, formalin-inactivated hepatitis A vaccine is highly protective against HAV. A booster dose is required 6-18 months later for full immunity. Passive Immunication (prophylaxis or post-exposure prophylaxis to Hepatitis A: Pooled human Immune Serum Immuno Globulin (ISIG) 0.02 mL/kg single IM dose is recommended for travelers to endemic areas or for household & sexual contacts with serologically confirmed hepatitis A person, to be administered as soon as possible within 2 weeks of exposure. The duration of protection appears to be dose related, with the 0.02 mL/kg dose providing protection for approximately 3 months and a 0.05 mL/kg dose providing protection for 4 to 6 months.

68) Cervical Dysplasia - Found on 25 yo for well women's exam. Also, A 26 yo healthy woman came in for examination (cin III in pap smear). Cervical dysplasia is a condition in which the cervical tissue does not grow normally, causing precancerous changes which can range from mild to moderate to severe. In the most severe cases it can lead to cervical cancer. Risk factors are thought to include: Human papilloma virus (HPV) which is spread sexually is known to greaten the risk of developing cervical

dysplasia and cancer. cigarette smoking & substance abuse; TB; DES exposure; Sex & pregnancy at an early age. Multiple lifetime sexual partners Screening for cervical cancer Regular Pap smears are recommended for all women who are or have been sexually active and who have a cervix. Testing should begin when the woman first engages in sexual intercourse. Adolescents whose sexual history is thought to be unreliable should be presumed to be sexually active at age 18. Pap smears should be performed at least every 1 to 3 years. Testing is usually discontinued after age 65 in women who have had regular normal screening tests. Women who have had a hysterectomy including removal of the cervix for reasons other than cervical cancer or its precursors do not require Pap testing. Management of minor Pap smear abnormalities Satisfactory, but limited by few (or absent) endocervical cells Endocervical cells are absent in up to 10% of Pap smears before menopause and up to 50% postmenopausally. Management. The Pap smear should either be repeated annually or only recall women with previously abnormal Pap smears. Unsatisfactory for evaluation Repeat Pap smear midcycle in 6-12 weeks. If atrophic smear, treat with estrogen cream for 6-8 weeks, then repeat Pap smear. Benign cellular changes Infection--Candida. Most cases represent asymptomatic colonization. Treatment is offered for symptomatic cases. The Pap should be repeated at usual interval. Infection--Trichomonas. If wet preparation is positive, treat with metronidazole (Flagyl), then continue annual Pap smears. Infection--predominance of coccobacilli consistent with shift in vaginal flora. This finding implies bacterial vaginosis, but it is a non-specific finding. Diagnosis should be confirmed by findings of a homogeneous vaginal discharge, positive amine test, and clue cells on saline suspension. Infection-herpes simplex virus. Pap smear has poor sensitivity but good specificity for HSV; positive smears usually are caused by asymptomatic infection. The patient should be informed of pregnancy risks and the possibility of transmission. No treatment is necessary, and the Pap should be repeated as for a benign result. Inflammation on Pap smear Mild inflammation on an otherwise normal smear does not need further evaluation. Moderate or severe inflammation should be evaluated with a saline preparation, KOH preparation, and gonorrhea and Chlamydia tests. If the source of infection is found, treatment should be provided, and a repeat Pap smear should be done every 6 to 12 months. If no etiology is found, the Pap smear should be repeated in 6 months. Persistent inflammation may be infrequently the only manifestation of high-grade squamous intraepithelial lesions (HGSIL) or invasive cancer; therefore, persistent inflammation is an indication for colposcopy. Atrophy with inflammation is common in post-menopausal women or in those with estrogen-deficiency states. Atrophy should be treated with vaginal estrogen for 4-6 weeks, then repeat Pap smear. Hyperkeratosis and parakeratosis. Parakeratosis is defined as dense nuclei within a keratin layer. When no nuclei are present, the cells are designated hyperkeratotic. Parakeratosis and hyperkeratosis occur as a reaction to physical, chemical, or inflammatory trauma, and it may clinically appear as leukoplakia. Benign-appearing

parakeratosis or hyperkeratosis requires only a repeat Pap test in 6 months. When this finding persists, colposcopy is indicated.

69)Location:ER A 17 yo with DKA. and NIDDM pt with acute exacerbation, Dx: Ketoacidosis. Also TYPE II DM - 70 yr black male, resistant DM. Screening appropriate for patients with: a) family history of DM (remember DM type I has a stronger correlation with genetic transmission, but DM II is the frequent encounter-90%) b) significant obesity c) recurrent or hard healing skin wounds, d) recurrent genital or UTI infections (especially yeast) pay special attention to nonSTD's UTI's in young man e) pregnancy related problems-gestational diabetes, infant larger than 9 lb (over 4kg) f) symptomatic patients with 3P symptoms (polyuria, polydipsia,weight loss-some include polyphagia too) may be dgn without further testing when random PG is greater than 200mg/dl Screening methods: -fasting plasma glucose140mg/dl -DM (only one measurement is cosidered dgn.now!) -positive oral glucose tolerance test 200mg/dl Different criteria for pregnancy: -screening at 24-28wks for all pregnant women! -glycosuria anytime in pregnancy -fasting PG105 mg/dl -OGT 150 mg/dl Diffuse abdominal tenderness and rigidity in young person -check glucose level& serum/urinary ketones Management: ALL DM patients should have: 1. Diet control-ask for dietetician consult in CCS. caloric intake for normoponderals ~35kcal/day, obese ~5-15kcal/day , food composition:carbohydrate 55-60%complex glucocide preferred, protein 10-20%, fat 25-30% limit alcohol use recomand cessation of smoking 2. physical exercise-moderate effort proved to reduce insulin needs This should control type II DM for a good period of time Monitoring of therapy: self assessment of patient require intensive education-self-monitoring blood glucose devices (SMBG) 3 times/day-weekly blood glucose measurement at office-evaluation of therapy with Hb A1c at after 2 months ketone urinary assay Pharmacologic therapy should be implied if: -patient is unable of self assessment -noncompliance with prescribed tx -failure to achieve a normoglycemic status after a reasonable period of time -patients presenting with diabetic coma, -inability to mantain dietary intake -infection(any fever) -Type I DM 3. Preventive care: a) vaccinations-consider pneumoccocal and influenza vaccine for all diabetics + normal vaccinations related to

age b) yearly measurement of lipidic profile c) monthly measurement of blood pressure d) foot care education 4. Pharmacologic measures: Type II DM a) oral antidiabetic drugs: -sulfonylureas: use rather short drugs as glipizide/glyburide ContraIndication: DM I, pregnancy, children-caution in hepatic or renal disease toxic reactions: skin rash, blood dyscrasias, cholestatic jaundice, Disulfuram-like effects with alcohol Drug interactions: Salicilates,Warfarin, Sulfonamides, Cloramphenicol, Methyldopa, Miconazole, MAOI-may potentiate the hypoglycemic effect of sulfonylureas -biguanide: metformin-good choice for patients displaying hyperlipidemia associated with DM (increase HDL); when used alone metformin does not produce hypoglycemia. ContraIndication: liver, renal,heart failure, pulmonary isf., pregnancy, alcoholism b) Insulin therapy learn at least 1 type of insulin from each class i.e. short-acting insulin: Regular-onset 0.251h/semilente 0.5-1 h medium acting:lente onset 1-4 h long acting:ultralente onset 3-8h Complications of insulin tx: -Hypoglycemia-could be life threating-educate the patient regarding the signs, instruct to have permanent access to sugar/chocolate -insulin allergy-erythema, induration, pruritus at inj. site-treat with purified human insulin -lipoatrophy/lipodistrophy change the site of injections Chronic complications of DM 1) Ophtalmic complications recommandations: obtain an ophtalmo consulation at the time of dgn for DM II,and at 5 years after onset in type Iyearly consultation after that a. FO exam may reveal diabetic retinopathy: nonproliferative-limited to retina-microaneurysms, hemorhages, exudates-cotton wool aspect; proliferative- neovascularisation b. visual disturbances: acute monocular loss-retinal detachment, retinal embolic infarction;bilateral loss usually stroke blurred vision-diabetic cataract 2) diabetic neuropathy: pain-amytriptiline/application of capsaicin sensory defficit/motor defficit-no tx autonomic neuropathy-a) postural hypotension-treat spmtm neurogenic bladder-recurent UTI's-intermitent catherisation 3) Diabetic nephropathy-the principal cause of morbidity and mortality in diabetics, give ACE I, treat aggresivelu UTI's Hyperlipidemia-hypercholesterolemia & hypertrygliceridemia frequent accompany diabetes-increase in plasma tryglycerides &VLDL-give gemfibrosil along with diet. IN DKA WHERE DO YOU MANAGE THE PATIENT, ER, ICU, INPATIENT? ICU till the patient has been stable on his old method of insulin administration , ie sub q DO YOU GIVE K & WHEN? Along with insulin, some give a bolus of insulin and then start infusion, some start with infusion... controversial but both acceptable.. DO YOU ORDER ABG's & WHEN?

STAT and q 2 hours

70) Location: Emergency room Vital signs: BP:90/60 mmHg , HR:128/min regular , Temp:100.0° F, R.R:30/min rapid and shallow C.C: Vomitings and abdominal pain. HPI: A 20-yr-old woman presents to E.R with 5 episodes of vomiting, abdominal pain, weakness and increasing drowsiness of one-day duration. During the last 2 months she has noticed increased thirst and increased urination. The abdominal pain is diffuse, 4-5/10 in severity, constant, non-radiating and there are no aggravating or relieving factors. Vomiting is non-bloody. She has no other medical problems. She has no known drug allergies. She is not on any prescription or over the counter medications. She is not a smoker or alcoholic, and denies IV drug abuse. She has a family history positive for Type 1 Diabetes Mellitus. Her father, and paternal uncle and grandfather are all diabetics. Review of systems: She denies weight changes, fever, chills, night sweats, diarrhea, constipation, skin, hair, or nail changes, blurry vision, acute bleeding, easy bruising, indigestion, dysphagia, changes in bowel movements, bloody stools, burning on urination, recent travel, ill contacts, vaginal discharge or itch, pregnancy, heat or cold intolerance, drug or alcohol use. Last menstrual period ended four weeks ago, was normal in flow and duration. How do you approach this case? First quickly examine the patient General HEENT Neck Heart Lungs Abdomen Extremities Here are the results of the exam: General: Patient is in mild to moderate abdominal pain and appears very distressed . HEENT: Very dry mucus membranes, no JVD, EOM are intact. Rest is unremarkable. Lungs: Clear to auscultation B/L. Heart: Completely normal except tachycardia. Abdomen: Soft, non tender, normal bowel sounds and no guarding or rigidity. Extremities: No edema, calf tenderness, but week peripheral pulses.

1.

Discussion: Now, make a mental checklist of differential diagnosis, i.e. Abdominal pathology like appendicitis, gastroenteritis, pancreatitis, acute intestinal obstruction etc. 2. Menstrual symptoms or pregnancy related complications 3. DKA (Based on the family history and presenting clinical features) 4. Nonketotic Hyperosmolar state 5. Alcoholic ketoacidosis 6. Drug intoxication

Order the following stat: Pulse oxy, stat and continuous Oxygen, nasal canula 2 lit, continuous NS 0.9%, bolus, stat (This patient is severely dehydrated. She is hypotensive and tachycardic. So, she needs IV fluids.)

NS 0.9%, continuous, stat Finger stick glucose test, stat Urine pregnancy test, stat CBC with differential, stat BMP, stat EKG, stat Serum amylase, stat Serum lipase, stat Blood alcohol, stat Blood acetaminophen, stat Urine toxicology screen, stat Abdomen KUB, stat U/A, stat Ok here are the results: Pulse oxymetry showed 96% on room air Finger stick glucose shows 600mg/dL Urine pregnancy test is negative WBC 10,000/µL and normal differential Sodium is 129, Potassium is 5.0, Chloride is 90, Co2 is 14, calcium is 8.0, and a blood sugar of 600mg/dL EKG sinus tachycardia, nothing concerning Serum Amylase - mildly elevated Serum Lipase WNL Serum alcohol not present Serum acetaminophen - Negative Urine tox screen is – Negative for substance abuse Abdomen KUB is negative for obstruction, and no intraabdominal pathology is seen U/A showed 4+sugar, but no evidence of infection How do you approach this case? So this patient most likely has either DKA or Non-ketotic hyperglycemia. The diagnosis is based on clinical features, elevated blood sugars, and increased anion gap. To confirm the diagnosis we need to order serum ketones and serum osmolality. She has pseudohyponatremia i.e. secondary to elevated blood sugars. Treatment of hyperglycemia resolves her hyponatremia. Now order: Stat serum osmolality, stat Serum ketones, qualitative, stat Give regular insulin 15 units (bolus), stat Followed by regular insulin, IV, continuous Put the patient on cardiac monitor ABG, stat Serum Phosphate levels Serum Mg levels Here are the results: Serum Osmolality 305 Serum Ketones - high Serum Phosphate 3.2 (WNL) ABG showed metabolic acidosis, compensated by respiratory alkalosis (pH of 7.3) Review orders: Admit the patient to the intensive care unit Nothing by mouth Bed rest Vitals as per ICU protocol Strict input and urine out chart Add potassium 20 - 30 meq to each liter of IV fluids HbA1C level Follow the patient with

1.

BMP Q 2-4 hours, then Q 8-12hours, then Q day 2. ABG Q 2 hoursx2

After 4 hrs 1. Stop 0.9% NS and give ½ Normal saline, IV, continuous Monitor potassium deficiency and add IV potassium chloride as needed Consider antibiotics if the precipitating cause is an infection, get a chest X-ray, sputum gram stain, and culture/sensitivity; obtain blood cultures, U/A and urine cultures. Once nausea is decreased, start oral fluids. Once the patient is stabilized transfer to ward/floor.

The ComputerUSMLE step 3 General information: This section is intended for those preparing for the USMLE Step 3 examination. We have tried to put all of the important information into one place, so it’s easily accessible for you. First of all, don't believe those who say the Step 3 is easy. It may be easy for AMG for a variety of reasons, but not so for IMGs. The exam is difficult, but doable. You should plan ahead and study as much as you can. How much time you need depends on your situation. If you have taken step-2 a few years ago and have not studied since, you probably need more time. If, on the other hand, you completed your step 2 a few weeks or months ago, then you need less time. Most applicants study between two-four months. After figuring out your timetable, you need to get a few good sources to study. There are lots of books, CDs, and study courses available for step 3. Obviously, you won’t have enough time to read all of them. You should select a couple of sources and stay with them. Most books or CDs give you the same information in different styles or formats, but the material is basically the same. You need some reference books to look things up. CMTD and Washington manuals are pretty good sources. CCS: The Computer Based Case Simulations (CCS) currently comprises about 25% of the Step 3 examination. At this time, it consists of ten cases with 20- 25 minutes assigned to each case. Examinees manage the case without prompting, using a variety of diagnostic and treatment options. As simulated time passes, the patient's status will change based on the response to your management decisions. Acute cases may need to be managed in a short period of time, while patients with chronic problems will require management over weeks or months of simulated time. Each case begins with a brief description of the patient's appearance and reason(s) for the visit. You may be following patients in both inpatient and outpatient settings. To do well on this part of the Step 3, it is imperative that you practice with the USMLE's CCS software. You can download the software at www.usmle.org You will be judged on the actions taken, their sequencing, and timing. One measure of your score will be whether the patient was subjected to unnecessary testing or therapy. Also, an important consideration and evaluation will be whether the patient was placed at serious risk as a result of your action or

failure to act. Evaluation emphasizes process rather than outcome.

View more...

Comments

Copyright ©2017 KUPDF Inc.
SUPPORT KUPDF